Public health Flashcards

1
Q

PBH - 1
Which phase of the public health cycle is based on epidemiological analyzes?
A) situation analysis
B) determining the intervention options
C) planning interventions
D) preparing a strategic plan
E) implementation of health promotion programs

A

ANSWER
A) situation analysis

EXPLANATION
The first phase of the public health cycle (the phase which is repeated regularly during the active cycle) is an epidemiological analysis of the populations’ health status (and the factors influencing it), which is attached to strict professional aspects, and the evaluation of which is called situation analysis. In this phase, it is often necessary to characterize the temporal changes of a given health indicator or a health determinant as well as to determine whether a change took place with respect to a reference value and if it did, to what extent and into which direction

How well did you know this?
1
Not at all
2
3
4
5
Perfectly
2
Q

PBH - 3
Whose activity can be related to the establishment of occupational health as a subdiscipline of public health?
A) Max Pettenkofer
B) Bernardino Ramazzini
C) Fodor József
D) Robert Koch
E) Rachel Carlson

A

ANSWER
B) Bernardino Ramazzini

EXPLANATION
The first professional book related to occupational health, Ramazzini’s work entitled ” De morbis artificum diatribe ” was published in 1700, and it has been considered to be a milestone in establishing the subdisciplines of public health

How well did you know this?
1
Not at all
2
3
4
5
Perfectly
3
Q

PBH - 4
When was the eradication of smallpox declared by the WHO?
A) 1967
B) 1979
C) 1993
D) 2003
E) only Europe is smallpox-free

A

ANSWER
B) 1979

EXPLANATION
In 1979, as a result of the smallpox eradication program organized and managed by the WHO, the organization declared that smallpox had been eradicated on a worldwide level, which served not only the overall acknowledgement of public health activity, but also boosted the development of public health.

How well did you know this?
1
Not at all
2
3
4
5
Perfectly
4
Q

PBH - 5
The weight of healthcare in determining the health status in developed countries is approximately:
A) 5-10%
B) 10-15%
C) 15-20%
D) 20-25%
E) 25-50%

A

ANSWER
B) 10-15%

EXPLANATION
The weight of the different determinants is not equal in ensuring health. Although the contribution of the individual factors is debated (and may indeed differ in different communities), it is generally accepted that the genetic factors and the quality of healthcare determine the state of health in approximately 15-30% and 10-15%, respectively, while the lifestyle factors that are closely related to socio-economic status and education are responsible for the remaining 55-75% in determining health.

How well did you know this?
1
Not at all
2
3
4
5
Perfectly
5
Q

PBH - 6
Which screening type implies newborn screening?
A) mandatory screening
B) targeted screening
C) mass screening
D) screening of groups with increased risks
E) diagnostic screening

A

ANSWER
C) mass screening

EXPLANATION
The term “mass screening” emphasizes that the given screening test covers the entire potentially affected population. Typical examples for mass screening are, for instance, newborn screening (e.g. for phenylketonuria), the screening of the fetus for detecting developmental disorders (e.g. by ultrasound examination, etc.).

How well did you know this?
1
Not at all
2
3
4
5
Perfectly
6
Q

PBH - 7
To evaluate the effectiveness of public health programs, the following is essential:
A) the knowledge of mortality
B) the knowledge of morbidity
C) the screening of the population
D) establishing and operating health monitoring systems
E) the exact knowledge of the impact of the interventions

A

ANSWER
D) establishing and operating health monitoring systems

EXPLANATION
Evaluating the effectiveness of public health programs is not possible without establishing and managing a health monitoring system. Establishing and managing registries (utilizing modern information technology) of communicable and non-communicable diseases (including circulatory, carcinomatosus, digestive, respiratory, neurological, mental and musculoskeletal diseases) as well as registries of accidents are necessary, but not sufficient. This activity should be accompanied by monitoring the factors influencing health status (lifestyle, social, economic and environmental factors).

How well did you know this?
1
Not at all
2
3
4
5
Perfectly
7
Q

PBH - 8
Which organization developed the Public Health Code of Ethics describing the valid ethical principles of public health activity?
A) World Health Organization (WHO)
B) American Public Health Association
C) United Nations Organization
D) European Public Health Association
E) An ad hoc committee established by the WHO for this purpose

A

ANSWER
B) American Public Health Association

EXPLANATION
The American Public Health Association (APHA) defined the Public Health Code of Ethics in 12 points in 2001.

How well did you know this?
1
Not at all
2
3
4
5
Perfectly
8
Q

PBH - 9
Which indicator is not part of the vital statistics based on civil registries?
A) fertility
B) migration
C) mortality
D) natality
E) reproduction

A

ANSWER
B) migration

EXPLANATION
Population dynamics - ongoing population processes - are characterized by vital events and migration over a specified period (usually a one year span). The basic elements of natural population dynamics are natality/birth rate, fertility, mortality and reproduction. Civil registries (vital statistics) are the main information source for vital events/population dynamics

How well did you know this?
1
Not at all
2
3
4
5
Perfectly
9
Q

PBH - 11
Which statement is true related to relative risk?
A) it compares the frequency of exposure among patients to the frequency of exposure among healthy subjects
B) it belongs to the category of difference indicators
C) it is a measure of the incidence observed among the exposed subjects to the incidence of unexposed population
D) all of the statements are true
E) none of the statements is true

A

ANSWER
C) it is a measure of the incidence observed among the exposed subjects to the incidence of unexposed population

EXPLANATION
Risk indicators are indicators which characterize the relationship between a givendisease and the incidence of an assumed risk factor. Their estimation is based on comparison of the incidences of two (or more) groups from the population, in which the groups’ risk factor’s exposure levels are different. The denominator of the comparison usually is the incidence of the disease among non-exposed population. Risk indicators may be either ratio indicators (relative risk) or difference indicators (attributable risk).Relative Risk (RR) is the incidence ratio among exposed and unexposed persons (RR=events when exposed/events when not exposed).

How well did you know this?
1
Not at all
2
3
4
5
Perfectly
10
Q

PBH - 12
Which attributable risk indicator depends on the prevalence of exposure among the population?
A) incidence of exposed
B) attributable risk
C) attributable risk fraction
D) population attributable fraction
E) all of the above

A

ANSWER
D) population attributable fraction

EXPLANATION
The attributable (AR) risk is the excess risk fraction of the exposured against non-exposured. AR = incidence among exposed - incidence among non-exposed. The attributable risk fraction (ARF) is the reduction in incidence that would be ovbserved in case of the exposition would cease in the entire population (ARF = incidence of the exposed - incidence of the non-exposed / incidence of the exposed). Attibutable risk indicators only indicate the excess risk caused by the risk factor itself and do not rely on prevalence (rate of exposure) of the risk factors in a given population. However, population attributable risk takes into account this feature: Population attributable risk = incidence in the entire population - incidence of non exposured

How well did you know this?
1
Not at all
2
3
4
5
Perfectly
11
Q

PBH - 14
Which statement related to odds-ratio is false?
A) it is the probability ratio of exposure among cases and controls
B) if the odds-ratio is exactly 1 then there is no relationship between the risk factors and the disease
C) it can be calculated from contingency tables
D) it can be calculated from case-control studies

A

ANSWER
A) it is the probability ratio of exposure among cases and controls

EXPLANATION
Calculation of odds ratio (OR) among case and control groups is based on the probability of exposition derived from the 2x2 contingency tables. The odds of exposition in the case group: a/c ; the odds of exposition in the control group: b/d, so the ratio of the two is the odds ratio. The odds ratio is 1 if there is no relationship between the potential risk factor and the disease.

How well did you know this?
1
Not at all
2
3
4
5
Perfectly
12
Q

PBH - 15
Which of the following is considered as late neonatal death?
A) death on the 5th day of life
B) death in the 5th week of life
C) death in the 5th month of life
D) all of them
E) none of them

A

ANSWER
E) none of them

EXPLANATION
During the first year after birth, distinct periods can be distinguished related to infant mortality based on age.These are the following: early infant (neonatal) death: 0-27 days (usually divided into 0-6 days - early neonatal, 7-27 days - late neonatal), late infant (postneonatal) death: 28th day - 1 year (the first birthday does not belong to the infant age).

How well did you know this?
1
Not at all
2
3
4
5
Perfectly
13
Q

PBH - 20
Which formula can be used to calculate the excess mortality rate? (expected number of deaths: E, observed number of deaths: O)
A) (O - E) / O × 100
B) (O - E) / E × 100
C) (E - O) / O × 100
D) (E - O) / E × 100
E) none of them are true

A

ANSWER
B) (O - E) / E × 100

EXPLANATION
The excess mortality rate is the difference between observed and expected number of deaths divided by the number of expected deaths. It can be converted to percentage form using the formula: (O - E )/E x 100; which describes how the percentage of deaths of the given population exceeds that of chosen for comparison base.

How well did you know this?
1
Not at all
2
3
4
5
Perfectly
14
Q

PBH - 21
Which formula is used to calculate relative excess mortality rate (REMR)? (expected number of deaths: E; observed number of deaths: O)
A) (O - E)/ O × 100
B) (O - E) / E × 100
C) (E - O) / O × 100
D) (E - O) / E × 100
E) none of them

A

ANSWER
A) (O - E)/ O × 100

EXPLANATION
The relative excess mortality rate and the excess mortality rate differ in their denominator only. The denominator of the relative excess death rate is the observed number of deaths instead of expected number of deaths. REMR=(O-E)/O ×100

How well did you know this?
1
Not at all
2
3
4
5
Perfectly
15
Q

PBH - 24
Standardized mortality ratio (SMR) can be calculated by indirect standardization using the following formula:
A) SMR = (observed number of deaths - expected number of deaths) × 100
B) SMR = (observed number of deaths - expected number of deaths) /observed number of deaths× 100
C) SMR = (observed number of deaths - expected number of deaths) / expected number of deaths × 100
D) SMR = observed number of deaths - expected number of deaths
E) none of them

A

ANSWER
E) none of them

EXPLANATION
Standardized mortality ratio (SMR, usually expressed as percentage) related to the given population calculated by indirect standardization is the ratio of the observed number of deaths and the expected number of deaths..(SMR = observed number of deaths/ expected number of deaths× 100)

How well did you know this?
1
Not at all
2
3
4
5
Perfectly
16
Q

PBH - 25
The true statement on mortality of Hungarian men’s aged 45-64 between 1991 and 1995 was:
A) higher than in 1981-1985
B) higher than in 2001-2005
C) higher than in 1920-1921
D) all of them
E) none of them

A

ANSWER
D) all of them

EXPLANATION
Analyzing the premature mortality of the Hungarian population stratified by age, we can point to the fact that the mortality of men aged 45-64 between 1991 and 1995 was worse compared to 1920-21. By 2006 this parameter decreased below the value observed in 1930-31.

How well did you know this?
1
Not at all
2
3
4
5
Perfectly
17
Q

PBH - 26
The ICD-10 classification…
A) has been valid since the 18th century.
B) uses an alphanumeric code system.
C) varies country by country.
D) the individual classes show overlaps.
E) quantifies mortality data.

A

ANSWER
B) uses an alphanumeric code system.

EXPLANATION
ICD-10 classifies diseases into standardized classes based on international criteria which were implemented in Hungary on January 1, 1996. The aim of the standardized unification is to ensure that the morbidity and mortality data derived from different sources and for different diseases be comparable. It uses an alphanumeric code system.

How well did you know this?
1
Not at all
2
3
4
5
Perfectly
18
Q

PBH - 27
Mortality burden of malignant tumors in Hungary:
A) is the most important related to premature adult mortality
B) takes the second place after cardiovascular diseases among premature adult deaths
C) is the most important related to old age mortality
D) has grown in the last decade
E) is not remarkabla in premature mortality among women

A

ANSWER
A) is the most important related to premature adult mortality

EXPLANATION
The mortality burden of malignant tumors is the most notable related to premature mortality (especially among women) and the second in total mortality (population age 0 to X) in Hungary. Cardiovascular diseases are the leading cause of death in old age mortality. The mortality of malignant tumors has decreased over the last decade

How well did you know this?
1
Not at all
2
3
4
5
Perfectly
19
Q

PBH - 29
The prevalence of hypertension among men aged 65-74 in Hungary is approximately:
A) 13-20%
B) 26-40%
C) 40-58%
D) 48-72%
E) 79-90%

A

ANSWER
D) 48-72%

EXPLANATION
The prevalence of hypertension among men aged 35-44 is 13-20% and 26-40% among men aged 45-54. It was 40-58% among men aged 55-64 and 48-72% among men aged 65-74 based on the data derived from General Practitioners Morbidity Sentinel Station Program (GPMSSP).

How well did you know this?
1
Not at all
2
3
4
5
Perfectly
20
Q

PBH - 30
The amount of the physiological daily sodium chloride requirement is approximately:
A) 1-2 g
B) 3-4 g
C) 5-6 g
D) 7-8 g
E) 9-10 g

A

ANSWER
A) 1-2 g

EXPLANATION
The physiological sodium chloride intake requirement is no more than 1-2 g per day which is exceeded even if we skip adding extra salt to our food. The WHO recommends that the daily salt intake should be kept below 5 g.

How well did you know this?
1
Not at all
2
3
4
5
Perfectly
21
Q

PBH - 32
Which is the most vulnerable population group related to osteoporosis?
A) children and adolescents
B) elderly women and men
C) workers exposed to permanent vibration exposure
D) women at postmenopausal period
E) athletes with increased physical activity

A

ANSWER
D) women at postmenopausal period

EXPLANATION
Decrease in bone mass is observed as the age progresses in both men and women, but the most significant decrease in bone mass can be seen among postmenopausal women. Osteoporosis is strongly related to the decrease of estrogen production

How well did you know this?
1
Not at all
2
3
4
5
Perfectly
22
Q

PBH - 34
Which individuals (related to their gender and family status) exhibit more complicated manifestation of reactive depression?
A) married men
B) divorced men
C) married women
D) divorced women
E) promiscuous individuals regardless of their gender

A

ANSWER
C) married women

EXPLANATION
Depression of married women is more severe than that of women who are not married based on the findings of cross-sectional studies.The opposite relationship has been observed in men, for example, married men are less likely to have depression and even if the depression has developed it shows milder symptoms.

How well did you know this?
1
Not at all
2
3
4
5
Perfectly
23
Q

PBH - 35
The most common causes of premature deaths related to fatal accidents are:
A) vehicle traffic accidents
B) falls
C) poisoning
D) burns
E) injuries related to firearms

A

ANSWER
A) vehicle traffic accidents

EXPLANATION
Approximately 40% of accidents causing deaths under 65 were caused by vehicle traffic accidents - especially motor vehicle accidents (men: 33.7%, women: 33.0%) - and every sixth death was caused by accidental falls in 2008. Above the age of 65 years, however, the weight of the two accidental death causes changes: accidental fall is the cause in approx.. two-thirds (59%) of accidental deaths in elderly men. The corresponding number among elderly women is 70.5%.

How well did you know this?
1
Not at all
2
3
4
5
Perfectly
24
Q

PBH - 36
Which of the two regions have the most increased risk of suicide-related deaths in Hungary?
A) Northern Great Plain and Southern Great Plain
B) Northern Great Plain and Northern Hungary
C) Southern Great Plain and Northern Hungary
D) Central Hungary and Northern Hungary
E) Central Hungary and Northern Great Plain

A

ANSWER
A) Northern Great Plain and Southern Great Plain

EXPLANATION
Traditional differencies can be seen related to suicide- mortality inside Hungary. In the Southern and Northern Great Plain the suicide related mortality of men aged 25-64 significantly (p<0.001; by 34% at lest) exceeded the Hungarian average suicide related mortality in the 1990s and 2000srespectively.

How well did you know this?
1
Not at all
2
3
4
5
Perfectly
25
Q

PBH - 37
The most influencing social factor related to suicide behavior is the following:
A) inability to loan repayment
B) residential segregation
C) existential uncertainty caused by economic crisis
D) entrepreneurial lifestyle
E) unemployment

A

ANSWER
E) unemployment

EXPLANATION
The most important social factor related to suicide behavior was unemployment according to a survey conducted in Hungary in 1995. Nearly 40% of the unemployed have suicidal thoughts and over 7% of them commit a suicide attempt which requires medical intervention (within the entrepreneurial group the corresponding numbers are 13.4% and 1.2%, respectively). The rate of repeated suicide attempts was highest among the unemployed, too.

How well did you know this?
1
Not at all
2
3
4
5
Perfectly
26
Q

PBH - 38
What does “phenotype prevention” mean?
A) Interrupting pregnancy (in the case of parental consent) to prevent giving birth to genetically impaired fetuses
B) Preventing the onset of a disease in individuals with increased risk genotype mainly by abolishing harmful environmental exposures
C) genetic counseling for couples with increased genotype risk who want to have children
D) gene therapy interventions among individuals to correct defective genes among individuals with increased genotype risk
E) estimation of population frequency of susceptible alleles related to diseases

A

ANSWER
B) Preventing the onset of a disease in individuals with increased risk genotype mainly by abolishing harmful environmental exposures

EXPLANATION
Detection of genetic predisposition can not be considered as a primary preventive intervention because the risk factor can not be excluded. It can not be considered as secondary preventive intervention either because it is not intended to help early detection of the disease but to detect the susceptibility. Detection of genetic predisposition of chronic non-communicable diseases is called phenotype prevention (as suggested and determined by Juengst in 1995) which falls between primary and secondary prevention. Phenotypic prevention’s main aim is to prevent the disease’s onset among endangered genotype individuals (high risk groups).

How well did you know this?
1
Not at all
2
3
4
5
Perfectly
27
Q

PBH - 40
Breast cancer’s incidence is mainly based on various mutations in the BRCA1 gene:
A) among Ashkenazi Jewish communities
B) among Hungarian female population
C) among Australian native population
D) in the background of breast cancer in elderly population
E) in the background of post-operative recidivation

A

ANSWER
A) among Ashkenazi Jewish communities

EXPLANATION
Susceptibility to various neoplastic diseases at population level is reflected in the different incidence rates that could be seen among different communities (geographically determined, based on ethnicity, etc For example, high incidence rate of breast cancer can be seen among Ashkenazi Jewish communities related to various mutations of the BRCA1 gene.

How well did you know this?
1
Not at all
2
3
4
5
Perfectly
28
Q

PBH - 41
Relative risk of lung cancer among smokers compared to non-smokers is approximately:
A) two times increased risk
B) five times increased risk
C) ten times increased risk
D) hundred times increased risk
E) 90% higher

A

ANSWER
C) ten times increased risk

EXPLANATION
The carcinogenic role of smoking is generally accepted based on the meta-analysis of epidemiological investigations. The relative risk of regular smokers for lung cancer is ten times higher compared to non smokers.

How well did you know this?
1
Not at all
2
3
4
5
Perfectly
29
Q

PBH - 44
The main aim of the North Karelia Project was:
A) to reduce cancer-related premature mortality
B) to reduce cardiovascular premature mortality
C) to reduce the mortality of mental illnesses
D) to improve the care of hypertensive patients
E) the early detection of cardiovascular diseases

A

ANSWER
B) to reduce cardiovascular premature mortality

EXPLANATION
The North Karelia Project was implemented by the Finnish Government in the early 1970s which main aim was to reduce the mortality rate of cardiovascular diseases. The project mainly focused on healthy eating and the reduction of smoking rate which resulted in a reduction of cancer-related mortality. Premature cardiovascular mortality decreased by 60%, and premature cancer-related mortality decreased by 40% within 25 years after implementing the Project.

How well did you know this?
1
Not at all
2
3
4
5
Perfectly
30
Q

PBH - 45
What does “hidden diabetes” mean?
A) no clinical symptoms are present due to treatment
B) not the primary disease but the complications of the primary disease cause the symptoms
C) the symptoms of the disease are masked by the characteristic symptoms of an other disease
D) only the hypertension accompanying diabetes is treated while the diabetes itself remains untreated
E) the existence of diabetes is unknown by both the patient and the doctor, so diabetes can be detected by a screening test only

A

ANSWER
E) the existence of diabetes is unknown by both the patient and the doctor, so diabetes can be detected by a screening test only

EXPLANATION
A study related to the observation of hidden diabetes was conducted among patients aged 55-64 based on the GPMSSP. According to the study, the prevalence of hidden diabetes is quite high, because every third or fourth diabetic patient was detected only by screening tests and not by the doctor or the patient.

How well did you know this?
1
Not at all
2
3
4
5
Perfectly
31
Q

PBH - 47
Which is not a component of metabolic syndrome?
A) hypertension
B) increased fasting blood glucose level
C) elevated triglyceride level
D) elevated HDL-C value
E) visceral obesity

A

ANSWER
D) elevated HDL-C value

EXPLANATION
The main components of metabolic syndrome are the hypertension, increased fasting blood glucose level, elevated triglyceride value and visceral obesity based on both the ATPIII and the IDF guidelines. Only decreased HDL-C level is considered to be abnormal.

How well did you know this?
1
Not at all
2
3
4
5
Perfectly
32
Q

PBH - 51
The relative risk of premature mortality related to respiratory diseases) in Hungary compared to EU15 average
A) is more than two times higher
B) is more than three times higher
C) is more than four times higher
D) does not differ significantly ()
E) is 20% lower

A

ANSWER
A) is more than two times higher

EXPLANATION
The relative risk of premature mortality related to respiratory diseases has been more than two times higher among both genders since 2005 in Hungary compared to EU15’s mortality. (for men, 2.8, for women 2.3 in 2007)

How well did you know this?
1
Not at all
2
3
4
5
Perfectly
33
Q

PBH - 52
Suicide mortality was characterized by the following over the past three decades in Hungary:
A) it has drastically increased
B) a temporary declining trend was followed by stagnation since the Millennium
C) more women are affected than men
D) mortality rates decreased significantly for both gender
E) the highest suicide mortality for both gender was observed in the Northern Hungary Region

A

ANSWER
D) mortality rates decreased significantly for both gender

EXPLANATION
A decreasing trend was observed in recent decades related to suicide mortality. Mortality of men aged 25-64 in 2008 was nearly 50% lower than in 1980, while mortality of women decreased by one third. Highest suicide rate can be seen in Northern Great Plain and Southern Great Plain.

How well did you know this?
1
Not at all
2
3
4
5
Perfectly
34
Q

PBH - 53
Utilization of the findings of genomic research in public health:
A) is free of charge and widely available
B) is limited
C) genetic screening can only be used after payment
D) a complete lack of legislation is missing to ensure their implementation
E) screening tests which could prove genetic predisposition to cancer are unreliable

A

ANSWER
B) is limited

EXPLANATION
The application of knowledge related to genetic / genomic science in public health is still limited. The obstacles hindering utilization are as follows. i) Genetic epidemiological studies were rarely conducted in potentially affected populations. ii) The applied methods are usually cost-intensive. iii) Preventive interventions are available only in a small minority of the identified risk conditions and the acceptance of these interventions for the individuals is also questionable. iv) Human resources are not granted either for genetic-based screening in the field of public health or for dissemination of goals and results. v) . The legal background for the regulation of genomic researches is restricted.

How well did you know this?
1
Not at all
2
3
4
5
Perfectly
35
Q

PBH - 54
Which one of the following is not a geohelmint?
A) Strongyloides stercoralis
B) Trichuris trichiura
C) Ascaris lumbricoides
D) Taenia solium

A

ANSWER
D) Taenia solium

EXPLANATION
Strongyloides stercoralis, Trichuris trichiura and Ascaris lumbricoides are geohelminths, while Taenia solium is not. Infection with Taenia solium and Taenia saginata occur through the consumption of inadequately cooked, semi-cooked or raw meat. The intermediate hosts for Taenia solium are swine and for Taenia saginata are cattles. The eggs could enter the body of the animals orally with infected human feces or with infected fodder They can get everywhere in the body via blood circulation, but they usually settle in the mucles. (cystycercus).

How well did you know this?
1
Not at all
2
3
4
5
Perfectly
36
Q

PBH - 55
What vaccine should be given to newborns of HbsAg-positive mothers?
A) passive immunization against hepatitis B
B) active immunization against hepatitis B
C) both
D) none of them

A

ANSWER
C) both

EXPLANATION
Newborns of HbsAg positive mothers should receive both active and passive immunization against hepatitis B. Protective antigen of HBV is HbsAg. HBs antibody provides protection against it. HBV can not be produced without hosting human body (in vitro). The protective antigen was produced by plasmapheresis from the plasma of healthy carriers in the past. Nowadays, it is produced using standard genetic engineering techniques, the virus’s HbsAg-encoding S gene has been implanted into Saccharomyces cerevisiae and can be produced under large-scale circumstances. Component of human origin is not present in the vaccine. Protective antibodies are produced from sera with high titers for passive immunization. Newborns should be given hyperimmun γ globulin within 12 hours as well as the iniation of active immunization should start. The latter must be followed by revaccination at month 1 and and 6.

How well did you know this?
1
Not at all
2
3
4
5
Perfectly
37
Q

PBH - 56
There is no active vaccination commercially available in Hungary against the following disease:
A) hepatitis B
B) influenza
C) typhus abdominalis
D) salmonellosis
E) yellow fewer

A

ANSWER
D) salmonellosis

EXPLANATION
Prevention of salmonellosis is not related to vaccination but it is a hygienic task. This task includes health monitoring of animals which are going to be processed that is the veterinarian’s job. Regulations of production, storage and transport of food are food hygiene tasks. Providing drinking water, proper sewage treatment and disposal, fly and pest control belong to environmental and community health. Personal hygiene is also quite important related to preventive services. Strict observance of hygienic rules is important during food preparation, storage, transportation and catering to prevent salmonellosis.

How well did you know this?
1
Not at all
2
3
4
5
Perfectly
38
Q

PBH - 58
Which one of the following in not included in age-related mandatory vaccination in Hungary?
A) vaccination against mumps
B) vaccination against rubella
C) vaccination against smallpox
D) vaccination against tetanus

A

ANSWER
C) vaccination against smallpox

EXPLANATION
Vaccination against mumps, rubella and tetanus belong to mandatory vaccination according to the vaccination schedule which are received in the form of combined vaccines. Vaccination against smallpox was mandatory up to 1980 (eradication of smallpox).

How well did you know this?
1
Not at all
2
3
4
5
Perfectly
39
Q

PBH - 59
How long must the food samples be stored in kitchens providing public catering services?
A) 6 hours
B) 12 hours
C) 24 hours
D) 48 hours
E) 72 hours

A

ANSWER
E) 72 hours

EXPLANATION
Food samples should be kept for 72 hours. Detecting the source of food-related infections and poisonings can be done by the investigation of food samples. Kitchens providing public catering - over 29 servings per day - have to store a 100g food sample of each meal in a clean, microbiologically and chemically acceptable container or packing bag and in a refrigerator for 72 hours. Contents, date of sampling and the name of the person performing the sampling must be attached to the sample. If bacteria was found in the patients’ feces and the bacteria’s antigenic structure could be found in the food sample, as well as the infection was proved by laboratory tests, then the infection developed because of the food consumption. If the symptoms of the suspected food consumers are similar and their latency time is almost the same then the relationship between food consumption and the bacteria is proved epidemiologically.

How well did you know this?
1
Not at all
2
3
4
5
Perfectly
40
Q

PBH - 61
In which sterilization method are carcinogenic compounds used?
A) dry heat sterilization
B) autoclaving
C) chemical sterilization
D) plasma sterilization

A

ANSWER
C) chemical sterilization

EXPLANATION
Liquids (formaldehyde) and gas (ethylene oxide) are used for chemical sterilization of heat-sensitive materials to destroy microorganisms. Their application allows lowering the sterilization temperature.but both compounds are known to have carcinogenic effects. During dry heat sterilization dry air with temperature between 160 and 200 oC circulates in a sealed space in which microorganisms will be inactivated, then destroyed due to high temperature. Autoclaves operate with saturated, pressurized water vapor. The microorganisms are destroyed by the heat energy released from the vapor condensed on the surface of the deposited materials. During plasma steriliztaion, the sterilization will be done in the hydrogen peroxide plasma which will split into water and oxygen

How well did you know this?
1
Not at all
2
3
4
5
Perfectly
41
Q

PBH - 62
Which of the following procedures is not used for sterilization?
A) UV light
B) γ-radiation
C) ethylene oxide
D) autoclaving

A

ANSWER
A) UV light

EXPLANATION
γ-radiation, ethylene oxide and autoclaving procedures are used for sterilization, while UV light is used in operating rooms, medical office waiting rooms and laboratories when the staff is not present. Timindimers are produced in the DNA of microorganisms during UV radiation, which penetration is weak so it can only be used for sterilizating surfaces and for disinfecting the indoor air.

How well did you know this?
1
Not at all
2
3
4
5
Perfectly
42
Q

PBH - 63
Infectious and suspected of being infectious patients detected in primary care should be reported to?
A) the local institution of inpatient care
B) the subregional public health institute competent at the regional location of the infection
C) the subregional public health institute competenet at the infected patients’residence
D) the National Public Health and Medical Office Service, National Center for Epidemiology

A

ANSWER
B) the subregional public health institute competent at the regional location of the infection

How well did you know this?
1
Not at all
2
3
4
5
Perfectly
43
Q

PBH - 64
Which of the following infections should be reported urgently?
A) dysenteria
B) typhus abdominalis
C) campylobacteriosis
D) amoebiasis

A

ANSWER
B) typhus abdominalis

EXPLANATION
Highly infectious diseases, e.g. typhus abdominalis, cholera, poliomyelitis, lyssa, pestis, etc. have to be reported immediately by phone or by e-mail to the competent county public health service and to the National Public Health and Medical Office Service, National Center for Epidemiology.

How well did you know this?
1
Not at all
2
3
4
5
Perfectly
44
Q

PBH - 65
For which infection must be the patient quarantined at an inpatient hospital unit?
A) typhus abdominalis
B) cholera
C) both of them
D) none of them

A

ANSWER
C) both of them

EXPLANATION
Limiting and separating the movement of the infectious source (infected patient) can prevent the infectious pathogens from spreading. Separation could be done in an inpatient hospital unit’s quarantine room especially when it is indicated by the patient’s health status, too.

How well did you know this?
1
Not at all
2
3
4
5
Perfectly
45
Q

PBH - 67
In which diseases should the persons in close contact with the infected patient undergo a 7-day epidemiological surveillance?
A) dysenteria
B) campylobacteriosis
C) typhus abdominalis
D) Hepatitis A

A

ANSWER
A) dysenteria

EXPLANATION
Epidemiological surveillance is used on persons who could be potential infectious sources due to their direct contact with the infected patient. Its’ duration is equal to the maximum latency of the infectious disease. The duration of the epidemiological surveillance is 7 days for dysenteria, for campylobacteriosis 5 days, 21 days for typhus abdominalis and 30 days for hepatitis A infection.

How well did you know this?
1
Not at all
2
3
4
5
Perfectly
46
Q

PBH - 68
Which statement related to poliomyelitis is not true?
A) Reportable disease
B) must be reported by telephone, by fax
C) laboratory test is not obligatory
D) Feces sample, irrigation fluid, liquor and blood samples must be sent to the Virology Department of National Public Health and Medical Officer Service

A

ANSWER
C) laboratory test is not obligatory

EXPLANATION
Poliomyelitis anterior acute disease is a reportable disease, which must be reported by telephone and fax to the National Public Health and Medical Officer Service and the National Center of Epidemiology. Laboratory tests are mandatory. Feces sample, irrigation fluid, 1-2 liquor samples and native blood must be sent to the Virology Department of National Public Health and Medical Officer Service which must be followed by the resending of new blood samples after 3 weeks for laboratory retests.

How well did you know this?
1
Not at all
2
3
4
5
Perfectly
47
Q

PBH - 69
Which statement related to salmonellosis is not true?

A) It is a reportable disease
B) It must be reported on via a “food contamination, food poisoing notification” sheet
C) The regional public health institute should be notified by telephone
D) Laboratory tests are mandatory
E) For the correct diagnosis blood sample should be sent to the regional laboratory before treatment

A

ANSWER
E) For the correct diagnosis blood sample should be sent to the regional laboratory before treatment

EXPLANATION
Salmonellosis is a reportable disease. If the infection has been transmitted by food, apart from the usual custom notification, the onset and the suspicion of infection must be urgently reported (by telephone, fax) to the relevant sub-regional public health institute, as well as in written form by filling the “food contamination, food poisoning notification” sheet. The disease is considered as group poisioning if the number of patients is between 5-30, if more than 30 are affected, then poisoning refers to a mass poisoning. In the latter case, the regional public health institution reports to the National Public Health and Medical Officer Service which informs the Medical Office Service and the National institute of Pharmacy and Nutrition. Patients are prohibited to work with food and beverages as well as in childcare facilities until the emission of pathogens is ceased. Laboratory tests of the feces are mandatory for diagnostic purposes while release examinations have to be carry out for those whose occupation is related to a child community of 0-3 years old, or work in a “dangerous position” or give breast milk.

How well did you know this?
1
Not at all
2
3
4
5
Perfectly
48
Q

PBH - 70
What is an inapparent infection?
A) disease without clinical symptoms
B) an infection with mild symptoms that can only be detected during an epidemic
C) all infections that are cured without any harm/lesions
D) any infection that does not generate laboratory changes/alterations

A

ANSWER
A) disease without clinical symptoms

EXPLANATION
Inapparent infection occurs when the number of pathogens entering the organism is low or the microbe’s pathogenicity is low. The immune system of the body can quickly overcome the infection, so it won’t generate any clinical symptom.

How well did you know this?
1
Not at all
2
3
4
5
Perfectly
49
Q

PBH - 72
What is the most likely disease in the case of “food poisoning” when the symptoms appear in a 6-48 hours period after the consumption of homemade ham and mayonnaise French salad, and the symptoms are: abdominal pain, diarrhoea, vomiting, fever?
A) Staphylococcal food poisoning
B) Salmonellosis
C) Botulism
D) Yersiniosis
E) Trichinellosis

A

ANSWER
B) Salmonellosis

EXPLANATION
The difference between the diseases with similar clinical symptoms is in their latency period. In a staphylococcal foodborne disease, the incubation period is usually between 2 to 6 hours, while 6 to 48 hours for salmonellosis and 12 to 36 hours for botulism. The latency period of Yersiniosis is 3-7 days. The incubation period is usually 7 to 10 days in trichinellosis.

How well did you know this?
1
Not at all
2
3
4
5
Perfectly
50
Q

PBH - 74
In which disease is not the swine the infectious source? (Which disease is not spread by the swine?)
A) Brucellosis
B) Tularemia
C) Listeriosis
D) Taeniasis
E) Trichinellosis

A

ANSWER
B) Tularemia

EXPLANATION
Swine is not infectious source of the Tularemia; its reservoirs could be certain rodents (mice, rats), cats, sheep, etc. Reservoirs as well as infectious source of Brucellosis (Brucella suis), listeriosis, Taenia (Taenia solium) and trichinellosis are the swine.

How well did you know this?
1
Not at all
2
3
4
5
Perfectly
51
Q

PBH - 75
If a person has received a complete immunization within 5 years against lyssa, what is the applicable active immunization protocol in the case of a suspected new infection?
A) vaccination on days 0, 7 and 21 (2-1-1 doses)
B) vaccination on days 0, 7 and 21 (1-1-1 doses)
C) vaccinaton on days 0, 3 and 7 (2-1-1 doses)
D) vaccination on days 0, 3 and 7 (1-1-1 doses)
E) vaccination on days 0 and 3 (1-1 dose)

A

ANSWER
E) vaccination on days 0 and 3 (1-1 dose)

EXPLANATION
If the exposed person has received a complete vaccination sequence within 5 years, in the case of a new infection his (her) immunity is sufficient to receive a partial vaccination series only for active immunization. This scheme is described in answser E. Answer A corresponds to the full immunization for those who have not been vaccinated or who have been vaccinated for more than 5 years. Answers B, C and D are incorrect, there are no vaccinatiol protocols like these.

How well did you know this?
1
Not at all
2
3
4
5
Perfectly
52
Q

PBH - 76
In which case is the hospital placement of the patient mandatory?
A) Anthrax
B) Leptospirosis
C) Listeriosis
D) Tularemia
E) Q fever

A

ANSWER
A) Anthrax

EXPLANATION
In order to provide an adequate treatment -adequate antibiotic therapy- for the severe, occasionally lethal disease (pulmonary anthrax), the patient should be taken to the hospital as soon as possible.

How well did you know this?
1
Not at all
2
3
4
5
Perfectly
53
Q

PBH - 77
Which animal (s) is/are the infectious source (s) of Choriomeningitis lymphocytica?
A) ruminants
B) swine
C) dog
D) rodents

A

ANSWER
D) rodents

EXPLANATION
LCM virus reservoirs could be rodents (domestic and field mice, hamsters, guinea pigs, etc.) as well as other animals (e.g. some monkey species) not mentioned here. Humans can be infected due to the consumption of food contaminated with urine and / or feces of these animals or due to inhaling dust containing the pathogens.

Lymphocytic choriomeningitis, or LCM, is a rodent-borne viral infectious disease caused by lymphocytic choriomeningitis virus (LCMV), a member of the family Arenaviridae, that was initially isolated in 1933. The primary host of LCMV is the common house mouse, Mus musculus.

How well did you know this?
1
Not at all
2
3
4
5
Perfectly
54
Q

PBH - 78
Which disease incidence has increased in our country (in Hungary) in the age group over 40 years?
A) Scarlet
B) diphtheria
C) pertussis
D) Haemophilus influenzae meningitis
E) tuberculosis

A

ANSWER
E) tuberculosis

EXPLANATION
The new cases of tuberculosis have risen in the age group over 40 year in Hungary. The morbidity of scarlet fever among the 3-5 year-olds is the highest; Diphtheria has occurred only in the form of imported cases in the recent decades; Pertussis is a disease of infants who have not received active immunization yet; Infection of H. influenzae in children under 5 years has been reported sporadically.

How well did you know this?
1
Not at all
2
3
4
5
Perfectly
55
Q

PBH - 79
Which disease does not require that people who are in direct contact with the infected patient be treated with chemoprophylaxis?
A) pertussis
B) scarlatina
C) diphteria
D) meningitis epidemica
E) H. influenzae meningitis

A

ANSWER
B) scarlatina

EXPLANATION
Due to the fact that the cause of scarlet fever (Streptococcus pyogenes) occurs in 5-10% of the population in general and in 20-30% of the population during an epidemic, the possibility of an infection is not restricted only to the patient, but may happen from others, too. Consequently, chemoprophylaxis of the contacts has no significant preventive effects and does not need to be used. In other diseases, as circulating bacteria are limited in the population, the chemoprophylaxis of the contacts - by blocking the development of the infection originating from the patient- is effectively preventive and should be used.

How well did you know this?
1
Not at all
2
3
4
5
Perfectly
56
Q

PBH - 81
How many days before the onset of the symptoms can infectivity persist in people infected with Lyssa?
A) 1-2 days
B) 3-4 days
C) 5-6 days
D) 7-10 days
E) 12-15 days

A

ANSWER
D) 7-10 days

EXPLANATION
The amount of virus required for infectivity is presented in the excretion of the infected person 7-10 days before the onset of symptoms, so those being in contact may be infected with an asymptomatic person

How well did you know this?
1
Not at all
2
3
4
5
Perfectly
57
Q

PBH - 82
Which disease could cause teratogenic changes in the fetus during pregnancy?
A) Ornithosis
B) Tularemia
C) Leptospirosis
D) Q fever
E) Toxoplasmosis

A

ANSWER
E) Toxoplasmosis

EXPLANATION
Due to transplacental infection in pregnant women carrying toxoplasma gondi the fetus is prone to severe teratogenic changes which could affect the central nervous system. The pathogens of the other listed diseases do not pass the placenta, so they have no teratogenic effect.

How well did you know this?
1
Not at all
2
3
4
5
Perfectly
58
Q

PBH - 83
The mean age of patients with vCJB is
A) 29-30 years
B) 35-40 years
C) 50-60 years
D) over 60 years

A

ANSWER
A) 29-30 years

EXPLANATION
The epidemiological feature of vCJD is that it occurs in young adults with an average age of approximately 30 years. This is one of the major differences between the new variant and the classic CJD; the latter one mainly affects the elderly generations.

How well did you know this?
1
Not at all
2
3
4
5
Perfectly
59
Q

PBH - 85
Reducing ozone concentration by 1% in the stratosphere will increase the incidence of skin cancer by:
A) 1%
B) 2%
C) 3%
D) 4%
E) 5%

A

ANSWER
B) 2%

EXPLANATION
A 1% reduction in ozone concentration, which plays an essential role in the uptake of harmful ultraviolet rays, increases the incidence of skin cancer by 2%. (As a result of this change, cataract incidence increases by 0.5%, too.)

How well did you know this?
1
Not at all
2
3
4
5
Perfectly
60
Q

PBH - 86
At which heavy metal exposure are people with hyperthyroidism not allowed to be employed/to work?
A) arsenic
B) mercury
C) lead
D) cadmium
E) chrome

A

ANSWER
B) mercury

EXPLANATION
People with hyperthyroidism should not be exposed to mercury. Non-serious thyroid gland disease does not exclude the employability with/of other heavy metals.

How well did you know this?
1
Not at all
2
3
4
5
Perfectly
61
Q

PBH - 87
Which of the following pesticides are persistent in surface waters?
A) organic phosphate esters
B) pyrethroids
C) carbamate
D) chlorinated hydrocarbon
E) dithiocarbamates

A

ANSWER
D) chlorinated hydrocarbon

EXPLANATION
Among the listed pesticides, chlorinated hydrocarbons (eg DDT) are persistent in surface waters, they decompose only over a long time. Compounds of the other four groups are decomposed in an aqueous environment in a short time. Organic phosphate esters are hydrolyzed in the presence of a water molecule, which results in the losing of their toxic effect.

How well did you know this?
1
Not at all
2
3
4
5
Perfectly
62
Q

PBH - 88
Which is a non-muscarine symptom?
A) Bradycardia
B) bronchospasm
C) increased sweating
D) spasm of the respiratory muscles
E) needle pupil

A

ANSWER
D) spasm of the respiratory muscles

EXPLANATION
The spasms of the respiratory muscles is a nicotine-like symptom due to organic phosphates, while bradycardia, bronchospasm, increased sweating and needle pupil are muscarinic alterations.

How well did you know this?
1
Not at all
2
3
4
5
Perfectly
63
Q

PBH - 89
Which of the following compounds has a human teratogenic effect?
A) toluene
B) cobalt
C) acrylamide
D) formaldehyde
E) methanol

A

ANSWER
A) toluene

EXPLANATION
Toluene has a teratogenic effect on humans; due to this special feauture, pregnant women are not allowed to be employed in case of toluene exposure.

How well did you know this?
1
Not at all
2
3
4
5
Perfectly
64
Q

PBH - 91
Which acute high-dose pesticide poisoning - with other severe symptoms - causes fever above 40° C?
A) carbamate
B) dithiocarbamate
C) phenoxycarboxylic acid derivative
D) Pyrethroid
E) dinitro-o-cresol

A

ANSWER
E) dinitro-o-cresol

EXPLANATION
High doses of dinitro-o-cresol may cause severe acute symptoms - tachycardia, respiratory disorders, spasms, lung oedema, accompanied by high fever above 40 ° C. Fever caused by acute dinitro-o-cresol intoxication cannot be cured with drugs effectively, but direct cooling of the patient (eg cold water bath) can reduce body temperature.

How well did you know this?
1
Not at all
2
3
4
5
Perfectly
65
Q

PBH - 92
Drinking water containing low level of magnesium will increase the incidence which of the following disease?
A) heart attack
B) cerebral stroke (dash)
C) stomach cancer
D) type 2 diabetes
E) colon cancer

A

ANSWER
A) heart attack

EXPLANATION
Based on epidemiological findings, out of the five diseases listed here only the positive relationship between myocardial infarct and the low level of magnesium in the drinking water is considered proven. (low magnesium consumption it increases the heart attack incidence)

How well did you know this?
1
Not at all
2
3
4
5
Perfectly
66
Q

PBH - 93
What does “dose toxica minima” mean?
A) The smallest amount of substance that is still responsible for eliciting the specific effects characteristic of the substance
B) The highest amount of substance to which the organism will not respond with lesions/abnormal alterations
C) Minimum amount of substance to which the body responds with non-typical changes that are not characteristic of the given substance
D) The smallest amount of substance that may have toxic effects
E) The smallest amount of substance to which the body responds with an effect typical of the substance

A

ANSWER
D) The smallest amount of substance that may have toxic effects

EXPLANATION
The minimum toxic dose (DT) is the smallest amount of a substance that already causes toxic effects, regardless of whether these effects reflect the general toxicity or are symptoms characteristic of the substance

How well did you know this?
1
Not at all
2
3
4
5
Perfectly
67
Q

PBH - 94
In which organic solvent exposure are dermatological patients not allowed to work?
A) benzene
B) toluene
C) xylene
D) carbon tetrachloride
E) gasoline

A

ANSWER
C) xylene

EXPLANATION
In the case of work related to xylene exposure - in addition to pregnant women, breastfeeding mothers, adolescents, alcoholics, people with hemopoietic and renal disease - dermatological patients are not allowed to be employed.

How well did you know this?
1
Not at all
2
3
4
5
Perfectly
68
Q

PBH - 95
In the case of acute poisoning which of the substances below can cause - after a few weeks latency - a reversible paralysis of the lower limbs?
A) formaldehyde
B) tri-ortho-cresyl phosphate
C) acrylonitrile
D) acrylamide
E) vinyl chloride

A

ANSWER
B) tri-ortho-cresyl phosphate

EXPLANATION
In the case of acute tri-ortho-cresyl phosphate intoxication after 2-4 weeks of the first symptoms (vomiting, diarrhea) paralysis of the lower limbs may occur that is usually reversible. The other compounds will not cause acute damage to the nerves of the lower limbs.

How well did you know this?
1
Not at all
2
3
4
5
Perfectly
69
Q

PBH - 97
The nitrate concentration limit of drinking water for infants to drink or their food to be prepared with is:
A) 10 mg / l
B) 20 mg / l
C) 30 mg / l
D) 40 mg / l
E) 50 mg / l

A

ANSWER
D) 40 mg / l

EXPLANATION
40 mg / l is the limit for the nitrate concentration in drinking water, above which the water is prohibited for babies to drink or to their food to be preapered with.

How well did you know this?
1
Not at all
2
3
4
5
Perfectly
70
Q

PBH - 99
Which of the following compounds is not excreted into the breast milk?
A) alcohol
B) nicotine
C) dioxin
D) PCB
E) phenol

A

ANSWER
E) phenol

EXPLANATION
Due to their lipophilic characteristic the first four substances can be excreted into the breast milk, so the breastfed child can be exposed to them. Phenol is hydrophilic, therefore it does not appear in the breast milk.

How well did you know this?
1
Not at all
2
3
4
5
Perfectly
71
Q

PBH - 100
The survival time of the geohelminths’ eggs in the soil is:
A) 1-2 months
B) 6-8 months
C) 12-15 months
D) 18-24 months
E) 2-3 years

A

ANSWER
E) 2-3 years

EXPLANATION
The resistance of geohelminths’ eggs is significantly greater than the resistance of not geohelminth worms’ eggs, thus they survive 2-3 years in the soil.

How well did you know this?
1
Not at all
2
3
4
5
Perfectly
72
Q

PBH - 102
Which is the organic solvent of which occupational exposure can be measured by the concentration of urine muconic acid?
A) benzene
B) toluene
C) xylene
D) ethylene glycol
E) Propyl alcohol

A

ANSWER
A) benzene

EXPLANATION
Determining the level of benzene exposure could be done by measuring the urine concentrations of one of its metabolites, muconic acid. Another possibility is the measurement of the dissolved product of benzene, the urinary phenol. In the case of toluene exposure, urinary hippuric acid or orthocresol concentrations can be measured, and the ortho hippuric acid content can be determined for xylene. The level of exposure to ethylene glycol and propyl alcohol can not be characterized by urinary metabolic measurement.

How well did you know this?
1
Not at all
2
3
4
5
Perfectly
73
Q

PBH - 103
Limit of nitrite concentration
A) 0.5 mg / l
B) 1 mg / l
C) 2 mg / l
D) 3 mg / l
E) 4 mg / l

A

ANSWER
A) 0.5 mg / l

How well did you know this?
1
Not at all
2
3
4
5
Perfectly
74
Q

PBH - 104
The chronic effect of which of the listed metals causes acrocyanosis?
A) lead
B) mercury
C) cadmium
D) arsenic
E) chromium

A

ANSWER
D) arsenic

EXPLANATION
Acrocyanosis is developed due to chronic arsenic exposure, its clinical manifestation is the “Blackfoot Disease”, which is a lower limb circulatory disorder.

How well did you know this?
1
Not at all
2
3
4
5
Perfectly
75
Q

PBH - 106
Which factor does not participate in the biological activity of the soil (geocycle)?
A) the filtering effect of soil particles
B) soil temperature
C) absorption
D) biological membrane
E) UV radiation

A

ANSWER
E) UV radiation

EXPLANATION
Physical (pelletization, moisture, temperature), physicochemical (absorption, precipitation) and biological (degradative organisms, biological membrane) factors participate in the biological activity of the soil. As the UV radiation does not penetrate the soil, thus it does not participate in the geocycling processes.

How well did you know this?
1
Not at all
2
3
4
5
Perfectly
76
Q

PBH - 107
Which compounds dissolve in the body during the toxic detoxification process?
A) toluene
B) freon
C) carbon tetrachloride
D) methanol

A

ANSWER
D) methanol

EXPLANATION
The process of toxic detoxification, namely a metabolism where the the degradation products are more toxic than the original molecule, is only observed related to methanol. Formaldehyde and formic acid, formed from methyl alcohol, can cause more severe symptoms (eg blindness) than the methanol itself. For the other four compounds (toluene, freon, carbon tetrachloride, benzene) the degradation products are less toxic than the original molecule.

How well did you know this?
1
Not at all
2
3
4
5
Perfectly
77
Q

PBH - 108
What is the minimum dose responsible for causing irreversible blindness in acute methanol intoxication?

A) 1-3 g
B) 8-10 g
C) 20-22 g
D) 25-30 g
E) 40-50g

A

ANSWER
B) 8-10 g

EXPLANATION
Consumption of 8 to 10 g of methyl alcohol causes irreversible blindness due to its direct effect on the optic nerve.

How well did you know this?
1
Not at all
2
3
4
5
Perfectly
78
Q

PBH - 109
Which compound is not involved in the effects caused by the Los Angeles-type smog?
A) aldehydes
B) ozone
C) sulfur dioxide
D) hydrogen peroxide
E) peroxyacetyl nitrate

A

ANSWER
C) sulfur dioxide

EXPLANATION
Sulfur dioxide is not involved in the effects of Los Angeles-type smog because it has no oxidative effect. However, all other compounds have such effect and, therefore, they actively participate in the induction of the symptoms of oxidative-type smog.

How well did you know this?
1
Not at all
2
3
4
5
Perfectly
79
Q

PBH - 110
The permissible cumulative annual equivalent dose of workers exposed to ionizing radiation is:
A) 5-10 mSv / year
B) 20 mSv / year
C) 50 mSv / year
D) 100 mSv / year
E) 500 mSv / year

A

ANSWER
B) 20 mSv / year

EXPLANATION
Occupational exposure level should be regulated so that the effective annual dose should not o-exceed 20 mSv (averaged over five consecutive years).

How well did you know this?
1
Not at all
2
3
4
5
Perfectly
80
Q

PBH - 111
Which monomer causes a sclerodermic change affecting the fingers of chemical plant workers?
A) vinyl chloride
B) acrylamide
C) acrylonitrile
D) vinylbenzene
E) phenylethylene

A

ANSWER
A) vinyl chloride

EXPLANATION
Vinyl chloride is a colorless, sweet smelling gas which causes exposition during the manufacturing and processing of PVC. Disorders of the nervous system, Raynaud’s syndrome, thrombocytopenia, fibrotic liver damage, pulmonary fibrosis, acro-osteolysis, and sclerodermoid lesions have been reported among workers exposed to long (about 15 years) exposition of moderate doses of Vinly chloride.

How well did you know this?
1
Not at all
2
3
4
5
Perfectly
81
Q

PBH - 112
High dose chronic or “long-term” exposure of which monomer is responsible for the liver hemangiosarcoma in chemical factory workers ?
A) acrylamide
B) vinylbenzene
C) vinyl chloride
D) phenylethylene
E) acrylonitrile

A

ANSWER
C) vinyl chloride

EXPLANATION
During the manufacturing and processing of PVC, the vinyl chloride which is a colorless, sweet smelling gas could be exposed to workers. High doses of chronic or long term exposition may cause liver hemangiosarcoma.

How well did you know this?
1
Not at all
2
3
4
5
Perfectly
82
Q

PBH - 115
It is forbidden to give one of the followings in the case of carbamate intoxication:
A) atropine
B) diazepam
C) toxogonin
D) other parasympathetic compounds
E) too much liquid

A

ANSWER
C) toxogonin

EXPLANATION
Carbamates are insecticides classified as toxic or highly toxic compounds based on their per os LD50 values. These organic phosphates are cholinesterase inhibitors, but their effect is reversible and lasts for short time so the enzyme reactivation occurs spontaneously. In case of acute poisoning, administration of reactivators (toxogonin) is prohibited (its intake will worsening the symptoms);otherwise the treatment is the same as described for organic phosphates.

How well did you know this?
1
Not at all
2
3
4
5
Perfectly
83
Q

PBH - 116
When is it recommended to remove the worker spraying with organich phosphoric acid ester from her/his job?
A) When the worker’s red blood cell AChE activity has decreased by 10% compared to the exposure-free self-activity.
B) When the worker’s plasma AChE activity decreased by 50% compared to its own basic activity.
C) When the worker’s red blood cell AChE activity has decreased by more than 50% compared to own baseline activity.
D) If the worker’s red blood cell AChE activity decreased by 20-50% from own basic activity.
E) Urgent action should be taken even in the case of the smallest decrease of the red blood cell AChE activity

A

ANSWER
C) When the worker’s red blood cell AChE activity has decreased by more than 50% compared to own baseline activity.

EXPLANATION
If the individual’s red blood cell cholinesterase activity is less than 50% compared to its own basic activity, then the person must be (permanently) removed from the position and this is also recommended when the enzyme activitiy decreased by 25% .

How well did you know this?
1
Not at all
2
3
4
5
Perfectly
84
Q

PBH - 117
What kind of radiation cause the cataracts of smeltery workers?
A) microwave radiation
B) visible light with high brightness
C) infrared radiation
D) stroboscopic effects

A

ANSWER
C) infrared radiation

EXPLANATION
Infrared radiation spans between the visible and the radiofrequency range of the electromagnetic spectrum. Besides the sunlight it is produced by processes and activities that use the thermal energy of infrared radiation.These activities include heating and dewatering processes, welding, glass manufacturing, drying and burning of coatings for public goods. Waves with wavelengths shorter than 2000 nm can damage the cornea, iris and lens.

How well did you know this?
1
Not at all
2
3
4
5
Perfectly
85
Q

PBH - 118
Industry or activity with a high risk of causing cancer is
A) rag-picking
B) pepper (paprika) splitting
C) iron and steel production
D) electrical installation

A

ANSWER
C) iron and steel production

EXPLANATION
High cancer-risk industries are those industrial areas, where most of the people employed are exposed to one or more carcinogenic materials and within this population of workers the incidence of tumorous diseases exceeds the expected incidence of cancer in the general population.

How well did you know this?
1
Not at all
2
3
4
5
Perfectly
86
Q

PBH - 119
Chromium and its compounds can cause:
A) mesothelioma
B) lung cancers
C) myeloma
D) lymphoma

A

ANSWER
B) lung cancers

EXPLANATION
Inhalation of the hexavalent form of chromium could cause cancers, e.g. bronchial and nasopharyngeal cancers (which should be monitored periodically).

How well did you know this?
1
Not at all
2
3
4
5
Perfectly
87
Q

PBH - 120
What is the effective temperature?
A) various combinations of air temperature and humidity which elicit the same heat sensation
B) various combinations of air temperature, humidity, and air movement which elicit the same heat sensation
C) combinations of air temperature and radiation that elicit the same heat sensation
D) combinations of air temperature and air flow that elicit the same heat sensation
E) none of them

A

ANSWER
B) various combinations of air temperature, humidity, and air movement which elicit the same heat sensation

EXPLANATION
Effective temperature is based on the relative humidity of the air as well as on the airflow rate, while the corrected effective temperature takes into account the heat radiation also. The effective temperature can be determined by a nomogram.

How well did you know this?
1
Not at all
2
3
4
5
Perfectly
88
Q

PBH - 122
Which is the lowest frequency where a noise-induced permanent increase in the hearing threshold can be detected?
A) 1000 Hz
B) 2000 Hz
C) 4000 Hz
D) 8000 Hz

A

ANSWER
C) 4000 Hz

EXPLANATION
An early hearing loss can only be detected by audiometrical investigations - initially in the range of 4000 to 8000 Hz. In Hungary a hearing loss at 4000 Hz that reaches/exceeds 30 dB (zajcsipke) has to be reported as increased exposition case.

How well did you know this?
1
Not at all
2
3
4
5
Perfectly
89
Q

PBH - 123
Which heavy metals chronic exposure causes allergies and contact eczema?
A) manganese
B) silver
C) nickel
D) iron

A

ANSWER
C) nickel

EXPLANATION
Nickel is used in electroplating and steel alloys, in batteries (Ni-Cd and Ni-hybrid) and as a chemical catalyst. Many goods are made of nickel-containing metals. Contact with the skin often causes contact dermatitis, eczema or asthma.

How well did you know this?
1
Not at all
2
3
4
5
Perfectly
90
Q

PBH - 124
The decibel means
A) the strength of the noise/sound.
B) the sound frequency.
C) the sound pressure.
D) the level of the sound pressure.

A

ANSWER
D) the level of the sound pressure.

EXPLANATION
During the measurement of sound pressure the hearing thereshold is taken as a reference and the pressure of the various sounds is compared to this value. The result is the logarithmic decibel scale that gives the sound pressure level.

How well did you know this?
1
Not at all
2
3
4
5
Perfectly
91
Q

PBH - 125
The pain threshold related to sound pressure level is:
A) 70 dB
B) 80 dB
C) 100 dB
D) 130 dB

A

ANSWER
D) 130 dB

EXPLANATION
During the measurement of sound pressure the hearing thereshold is taken as a reference and the pressure of the various sounds is compared to this value. The result is the logarithmic decibel scale that gives the sound pressure level.The hearing range spans from 0 dB (this corresponds to the hearing threshold of 20 μPa) to 120-140 dB (this is 2 × 107 μPa). Sound pressure greater than the pain threshold (approx.. 130 dB) causes only pain.

How well did you know this?
1
Not at all
2
3
4
5
Perfectly
92
Q

PBH - 126
Which substance is not asbestos?
A) crocidolite
B) Amosite
C) antofilit
D) tremolos
E) stalactite

A

ANSWER
E) stalactite

EXPLANATION
There are two known groups and six types of minerals related to asbestos: serpentine group - chrysotile, amphibole group -, crocidolite, amosite, antofilite, tremolite, actinolite.

How well did you know this?
1
Not at all
2
3
4
5
Perfectly
93
Q

PBH - 127
Which industrial activity has no risk of causing cancer?

A) aluminum production
B) furniture production
C) shoe making
D) coke production
E) working with silicate

A

ANSWER
E) working with silicate

EXPLANATION
High cancer-risk industries are those industrial areas, where most of the people employed are exposed to one or more carcinogenic materials and within the population of workers the incidence of tumor diseases exceeds the expected incidence that is the incidence of cancer in the general population.

How well did you know this?
1
Not at all
2
3
4
5
Perfectly
94
Q

PBH - 128
Which compound is not a proven carcinogen in humans?
A) aflatoxins
B) arsenic
C) benzene
D) cyclophosphamide
E) cyclosporine

A

ANSWER
E) cyclosporine

EXPLANATION
Aflatoxin, arsenic, benzene, cyclophosphamide are proven human carcinogens.

How well did you know this?
1
Not at all
2
3
4
5
Perfectly
95
Q

PBH - 129
The mechanism of action of carbamate pesticides
A) the inhibition of acetylcholinesterase
B) Inhibition of glutathione peroxidase
C) inhibition of C-SH group enzymes
D) inhibition of ATPase

A

ANSWER
A) the inhibition of acetylcholinesterase

EXPLANATION
Carbamates, like organic phosphates, are acetylcholinesterase inhibitors, but their activity is reversible and short acting, so the enzyme reactivation occurs spontaneously.

How well did you know this?
1
Not at all
2
3
4
5
Perfectly
96
Q

PBH - 130
DDT is:
A) phenolic fungicide
B) carbamate insecticide
C) chlorinated hydrocarbon
D) organic phosphate ester compound

A

ANSWER
C) chlorinated hydrocarbon

EXPLANATION
DDT is the most commonly known representative of chlorinated hydrocarbons.

How well did you know this?
1
Not at all
2
3
4
5
Perfectly
97
Q

PBH - 131
The first symptom of chronic mercury poisoning is:
A) paralysis of radial nerve
B) paralysis of ulnar nerve
C) small wave tremor
D) visual impairment

A

ANSWER
C) small wave tremor

EXPLANATION
Chronic exposure of mercury is most commonly caused by mercury vapor. Symptoms of the peripheral nervous system are dominating in chronic exposure of mercury, the so-called small- wave tremor mercurialis is especially characteristic, which starts in the facial and hand muscles (writing test) but eventually affects the whole body

How well did you know this?
1
Not at all
2
3
4
5
Perfectly
98
Q

PBH - 132
Polychlorinated biphenyls induce (activate?):
A) microsomal enzymes of the liver
B) glutathione peroxidase
C) superoxide dismutase
D) catalase

A

ANSWER
A) microsomal enzymes of the liver

EXPLANATION
Absorbed PCBs induce changes in steroid metabolism and metabolic processes due to the induction of microsomal enzymes in the liver.

How well did you know this?
1
Not at all
2
3
4
5
Perfectly
99
Q

PBH - 133
Early hearing loss caused by noise is indicated on the audiogram
A) a monaural recess (30 dB) seen at 8000 Hz
B) binaural negative spike at 4000 Hz (30 dB)
C) bilateral recess formation at any frequency
D) unilateral recess formation at any frequency
E) recess formation only at the 600 Hz (30 dB)

A

ANSWER
B) binaural negative spike at 4000 Hz (30 dB)

EXPLANATION
An early hearing loss can only be detected by audiometrical investigations - initially in the range of 4000 to 8000 Hz. In Hungary a hearing loss at 4000 Hz that reaches/exceeds 30 dB (recess, negative spike) has to be reported as increased exposition casebut the hearing loss is irreversible in these cases.

How well did you know this?
1
Not at all
2
3
4
5
Perfectly
100
Q

PBH - 135
Is the child (infant?) mortality differerent among the highest and lowest socioeconomic groups, respectively?
A) Yes, its the highest among infants with the highest status
B) yes, its the highest among infants with the lowest status
C) No
D) No general conclusion could be drawn, it varies from country to country.

A

ANSWER
B) yes, its the highest among infants with the lowest status

EXPLANATION
The socio-economic situation is a factor that has a significant impact on the health status and on its various indicators, including infant mortality. This factor tends to be more favorable in children with better socioeconomic status in all countries (resulting in smaller infant mortality rate).

How well did you know this?
1
Not at all
2
3
4
5
Perfectly
101
Q

PBH - 136
What changes could be seen related to the average life expectancy of men over the age of 65 in the different social strata/categories between 1972 and 2001 in England?
A) Increased in all social categories, but the increase was the highest among those in the highest category.
B) Increased in all social categories at similar rates
C) Increased in all social categories, but the change was higher in the lowest category compared to change affecting those in the highest social category
D) Increased in the highest category and decreased in the lowest category

A

ANSWER
A) Increased in all social categories, but the increase was the highest among those in the highest category.

EXPLANATION
The socio-economic situation is a factor that significantly affects health status and its various indicators, including life expectancy.Life expectancy was significantly greater among those in better position compared to those in lower socioeconomical groups in England.

How well did you know this?
1
Not at all
2
3
4
5
Perfectly
102
Q

PBH - 138
How can fetal alcohol syndrome be prevented?
A) if the mother ignores alcohol consumption during the first trimester of pregnancy
B) if the mother ignores alcohol consumption during the first two trimesters of pregnancy
C) if the mother ignores alcohol consumption throughout the pregnancy
D) it can not be prevented as is due to genetic predisposition

A

ANSWER
C) if the mother ignores alcohol consumption throughout the pregnancy

EXPLANATION
The best way to prevent fetal alcohol syndrome is to avoid alcohol consumption throughout the pregnancy, as the quantity of alcohol that would not cause harm to the fetus during pregnancy is not known.

How well did you know this?
1
Not at all
2
3
4
5
Perfectly
103
Q

PBH - 140
Who disposesthe alpha fetoprotein (AFP) test of a pregnant woman, when and for what purpose?
A) The pysician will dispose the test on the 12th week for the screening of the closure dosorders of the neural tube
B) The pysician will dispose the test on the 16th week for the screening of the closure dosorders of the neural tube
C) The health visitor will dispose the test on the 16th week for the screening of the closure dosorders of the neural tube.
D) The health visitor will dispose the test on the 18th week for the screening of the closure dosorders of the neural tube.

A

ANSWER
B) The pysician will dispose the test on the 16th week for the screening of the closure dosorders of the neural tube

EXPLANATION
According to the decree 33/1992 regarding the prenatal care of pregnants, serum alpha-fetoprotein test is performed during the 16th week of pregnancy. The test is ordered by the physician who is responsible for carrying the prenatal care.

How well did you know this?
1
Not at all
2
3
4
5
Perfectly
104
Q

PBH - 141
What is the most common cause of deaths among 10-18 year olds?
A) accidents
B) suicides
C) upper respiratory tract infections
D) gastrointestinal infections

A

ANSWER
A) accidents

EXPLANATION
Approximately 37% of deaths are caused by accidents among individuals aged 10-18. The number of deaths related to accidents is more then two times bigger among boys compared to girls.

How well did you know this?
1
Not at all
2
3
4
5
Perfectly
105
Q

PBH - 143
At what age do teenagers start smoking at the highest rate?
A) at the age of 10-11
B) at the age of 11-12
C) at the age of 13-15
D) at the age of 15- 18

A

ANSWER
C) at the age of 13-15

EXPLANATION
According to the 2009/2010 date of the HBSC study (Health Behaviour of School-age Children), the frequency of smoking in 11-year-olds seems low, but it’s increased more than 10 times among 15+ teenagers in European countries.

How well did you know this?
1
Not at all
2
3
4
5
Perfectly
106
Q

PBH - 144
Which illegal drug is consumed the most among adolescents in Hungary?
A) heroin
B) cocaine
C) extasy
D) marijuana

A

ANSWER
D) marijuana

EXPLANATION
According to data obtained from the National Drug Collection and Contact Center, the most commonly used illegal drug among adolescents is the marijuana. According to the European Monitoring Center for Drugs and Drug Addiction in Europe and according to the United Nations Office on Drugs and Crime (UNODC) worldwide marijuana is the most commonly used drug among teenagers.

How well did you know this?
1
Not at all
2
3
4
5
Perfectly
107
Q

repeated

PBH - 144
Which illegal drug is consumed the most among adolescents in Hungary?
A) heroin
B) cocaine
C) extasy
D) marijuana

A

ANSWER
D) marijuana

EXPLANATION
According to data obtained from the National Drug Collection and Contact Center, the most commonly used illegal drug among adolescents is the marijuana. According to the European Monitoring Center for Drugs and Drug Addiction in Europe and according to the United Nations Office on Drugs and Crime (UNODC) worldwide marijuana is the most commonly used drug among teenagers.

How well did you know this?
1
Not at all
2
3
4
5
Perfectly
108
Q

PBH - 145
Basic pediatric care is accomplished by
A) home paediatricians, school doctors
B) home paediatricians, school doctors, school dentists
C) home pediatricians, health visitors, school doctors, school dentists,
D) family doctors, home pediatricians, health visitors, school doctors, school dentists

A

ANSWER
D) family doctors, home pediatricians, health visitors, school doctors, school dentists

EXPLANATION
Basic medical care for children between the ages of 0 and 18 includes school health care in Hungary; the control of growth and the screening tests must be performed by family doctors, home pediatricians, school doctors, school dentists, and health visitors.

How well did you know this?
1
Not at all
2
3
4
5
Perfectly
109
Q

PBH - 146
Which disease is the leading cause of death among 65-74-year olds in Hungary?
A) circulatory diseases
B) malignant tumors
C) accidents
D) respiratory tract infections

A

ANSWER
A) circulatory diseases

EXPLANATION
according to mortality rates, deaths from circulatory diseases are more common among the Hungarian 65 to 74 year-olds compared to individuals over the age of 85 in the EU-15 countries (according to 2009 data: men: 44.5 %, Women: 44.6%).

How well did you know this?
1
Not at all
2
3
4
5
Perfectly
110
Q

PBH - 147
Where is the number of suicide-related deaths the highest among individuals over 65?
A) In the capital (Budapest)
B) in the Northern and Southern Great Plains
C) in Northern Hungary
D) in Western Transdanubia

A

ANSWER
B) in the Northern and Southern Great Plains

EXPLANATION
The highest suidice rates could be observed among men and women over the age of 65 years in the two regions of the Great Plain.

111
Q

PBH - 148
What is the most common cardiovascular disease among 65+ in Hungary?
A) unstable angina
B) high blood pressure
C) stroke
D) heart attack

A

ANSWER
B) high blood pressure

EXPLANATION
According to data obtained from the Hungarian General Practitioners’ Morbidity Sentinel Stations Program (GPMSSP), a very high incidence o high blood pressure (60-70%) can be seen among Hungarians aged 65+.

112
Q

PBH - 149
What is the definition of preterm birth?
A) birth before the termination of the 40th week during pregnancy
B) birth weight below 2500 g
C) birth weight below 3000 g
D) birth before the termination of the 37th week during pregnancy

A

ANSWER
D) birth before the termination of the 37th week during pregnancy

EXPLANATION
According to an earlier definition of the World Health Organization (WHO), premature birth was defined as the birth before the termination of the 37th pregnancy week or when the birth weight is below 2.500 g. However, the current WHO definition refers to premature birth when the birth take place before the end of the 37th week.

113
Q

PBH - 150
Which age and gender has the highest suicidal risk in Hungary?
A) men under 65 years
B) men over 65 years
C) women under 65 years
D) women over 65 years

A

ANSWER
B) men over 65 years

EXPLANATION
The risk of suicide in men over the age of 65 is the highest in Hungary; According to data of 2009, suicide rate was more than two and a half times higher than the EU average of the same age group and more than twice as high as the rate of deaths from suicide among Hungarian men under the age of 65

114
Q

PBH - 151
Which vitamins are extremely important for the elderly?
A) Vitamin B12
B) Vitamin B12, folic acid
C) Vitamin B12, Vitamin D
D) Vitamin B12, vitamin D, folic acid

A

ANSWER
D) Vitamin B12, vitamin D, folic acid

EXPLANATION
Sufficient intake of calcium and vitamin D in combination with exercise is a key factor in the prevention of osteoporosis and resulting bone fractures. Intake of adequate amounts of vitamin B12 and folic acid is necessary not only to prevent anorexia, but also to prevent the symptoms of neuropathy. The higher concentration of folic acid in plasma correlates with the improved cognitive performance observed among older people

115
Q

PBH - 152
What type of exercise would you primarily recommend to an elderly patient with arthritis?
A) cycling
B) running
C) gardening
D) walking

A

ANSWER
D) walking

EXPLANATION
The recommended exercise should be the walking, which duration and speed can be adjusted to the capacity of the patient and could be increased slightly with time. It wont put a great burden on the joints, it improves the patients’ endurance, no tools are required and can be easily integrated into everyday activities.

116
Q

PBH - 154
The most important risk factor of learned helplessness:
A) prematurity
B) positive family atmosphere
C) child abuse
D) chronic disease

A

ANSWER
C) child abuse

EXPLANATION
The learned helplessness develops through a sustained, prolonged loss of control, it includes the feeling of hopelessness, self-abandonment,and has other consequences, too (eg.decreased immune function, learning deficit.

117
Q

PBH - 155
Which tobacco ingredient is responsible for the development of smoking-related vascular diseases?
A) nicotine
B) polycyclic aromatic hydrocarbons
C) carbon monoxide
D) ammonia

A

ANSWER
A) nicotine

EXPLANATION
The cardiovascular complications due to smoking are related to the activity of stimulant nicotine, which acts as an acetylcholine agonist.

118
Q

PBH - 156
The importance of community interventions in health promotion is that
A) individual behavior is strongly influenced by family and friends
B) individual behaviour is determined by family and friends
C) designing community level interventions is easier than designing individual level interventions
D) collaboration with communities is less expensive than interventions targeting individuals

A

ANSWER
A) individual behavior is strongly influenced by family and friends

EXPLANATION
Human communities have a significant concious and non-concious impact on the behavior of their members. Individual behaviours which differ from the expected/acted behaviour of the community are not sustainable and, if maintained, often lead to social isolation. Thus, behaviour having a positive effect on health status can be most efficiently developed at community level by helping communities to increase their health awarness.

119
Q

PBH - 157
In the context of health education
A) We use purposeful learning opportunities to transfer knowledge on health-related information
B) we can create an internet portal
C) we can use personalized advice
D) we can organize an advertising campaign
E) all of them could be applied

A

ANSWER
E) all of them could be applied

EXPLANATION
Health education’s aim is to increase the individuals’ health-related knowledge and skills by creating learning opportunities with adequate methods (individual and group meetings, live and recorded lectures, printed and electronic communication, websites).

120
Q

PBH - 158
Select the normal BMI range.
A) under 25
B) from 20 to 24.9
C) from 18.5 to 24.9
D) over 25

A

ANSWER
C) from 18.5 to 24.9

EXPLANATION
The body mass index calculation formula is the following: body weight in kilograms has to be divided by the square of height in meters. According to the classification of the World Health Organization, the normal range is between 18.5-24.9. Those with BMI of 20-25 have the lowest risk of premature death. Under 20, the risk is increased in the industrialized countries. It seems that BMI below 20 does not play a causative role in higher mortality, but in this group the proportion of those who lost weight as a result of several diseases is higher. The BMI of Asian populations tends to be lower, consequently here a BMI value between 18.5-20.0 does not indicate an increased risk.

121
Q

PBH - 159
Which vitamin plays an essential role in iron absorption?
A) Vitamin A
B) Vitamin B12
C) Vitamin C
D) Vitamin D.

A

ANSWER
C) Vitamin C

EXPLANATION
The hem-iron absorbtion derived from hemoglobin and myoglobin found in meat is hihgly effective. The absorption of inorganic non-hem iron is considerably worse, and is influenced by nutritional factors. The absorption of non-hem iron is inhibited by phytates, polyphenols and oxalates found in plant foods. High calcium intake also inhibits iron absorption. Vitamin C and fermented foods facilitate iron absorption. The role of vitamin C in iron absorption is so important that it can be considered as the physiological role of this vitamin. Each meal should contain at least 25 mg of Vitamin C, if iron-absorbtion inhibitors are present, even more. There are slight differences in the absorption of hem-iron, but the differences in the absorption of non-hem-iron could be as high as 20 times.. In the case of mixed diet, the absorbtion of the total iron content is approx.. 10%.

122
Q

PBH - 160
The best way to prevent the development of folic acid deficiency diseases:
A) folic acid supplement
B) mandatory enrichment of bread flour with folic acid
C) voluntary fortification of folic acid for certain cereal products
D) varied diet

A

ANSWER
B) mandatory enrichment of bread flour with folic acid

EXPLANATION
Folic acid is a general methyl donor involved in the synthesis of nucleic acids and amino acids as an enzyme cofactor. In the absence of folic acid, severe disorders in the homocysteine-methionine metabolism and damage of the cell proliferation occur. Folic acid deficiency during pregnancy cause an increased risk of closure abnormalities of the neural tube as well as fetal growth retardation, resulting in low birth weight. To prevent the consequences of hyperhomocysteinemia, it is recommended to consume folic acid fortified foods for the whole population. In order to reduce neural tube closure disorders in recent years fortification of flour with folic acid is mandatory in 60 countries, regarding the fact that almost half of the pregnancies are not anticipated and, consequently, the folic acid supplementation may start late.

123
Q

PBH - 161
The prevalence of iodine deficiency is the lowest
A) in America
B) in Europe
C) in Africa
D) in Southeast Asia

A

ANSWER
A) in America

EXPLANATION
Iinadequate iodine supplement is the lowest in America (10%) and the highest in Europe (50%) in the general population. Although the effective anti-iodine preventive intervention, iodising the salt was estabilished in the 1920s, in Europe, even today less than half of the households use iodised salt. Despite the efforts of recent decades, the inadequate intake of iodine remains a major public health problem in Europe, althought the severe mental impairment related to iodine deficiency is very rare.

124
Q

PBH - 162
Too much iodine intake:
A) causes hypothyroidism
B) can induce hyperthyroidism
C) causes cretenism
D) causes obesity

A

ANSWER
B) can induce hyperthyroidism

EXPLANATION
Too much iodine intake can cause hyperthyroidism; especially in the long-defective areas, it was found that at the beginning of iodine supplementation thyrotoxicosis rarely occured (so-called iodo-Basedow disease) in individuals over 40. Iodine is an essential component of thyroid hormones. In the absence of iodine growth- and developmental disorders, deafness, damage to the nervous system, slowing of mental activity, apathy could develop. Severe iodine deficiency during pregnancy and during early childhood causes developmental disorders and severe psychomotor disoerders. Cretens are characterized by severe mental retardation. Increasing metabolism due to hyperthyroidism can lead to weight loss.

125
Q

PBH - 164
The risk of stomach cancer is increased by:
A) excessive alcohol consumption
B) insufficient intake of dietary fibers
C) consumption of very hot drinks
D) frequent consumption of salt-preserved foods

A

ANSWER
D) frequent consumption of salt-preserved foods

EXPLANATION
Frequent consumption of salted foods is associated with an increased risk of developing gastric cancer. The significant decrease in the incidence of gastric cancer in recent decades is due to changes seen in the preservation methods (traditional preservation methods was replaced by refrigeration) in the industrialized countries. Frequent consumption of very hot drinks causes an increased risk of developing esophageal cancer. Inadequate intake of dietary fibers and excessive alcohol consumption correlates with the development of colorectal cancer, not with gastric cancer.

126
Q

PBH - 165
The recommended maximum fat intake as a percentage of total energy intake according to WHO is approximately
A) 5%
B) 10%
C) 15%
D) 30%

A

ANSWER
D) 30%

EXPLANATION
It has long been assumed that it does not make any particular difference what part of our energy needs is covered by proteins, fats and carbohydrates. Epidemiological studies of recent decades have clearly confirmed not only the importance of individual nutrients but also their correct proportions. According to the WHO recommendations, 15-30% of the energy intake should come from fat consumption. The increase of fat intake is mainly due to the consumption of food from animal origins (meat, milk), which is at the expense of the complex carbohydrate consumption. These so-called high-energy, western-type diets increase the risk of many chronic non-infectious diseases.

127
Q

PBH - 166
Aflatoxins produced by the Aspergillus mold genotypes that contaminate food:
A) have estrogenic effects
B) have an antimicrobial effect
C) cause vomiting and diarrhea
D) can cause liver cancer

A

ANSWER
D) can cause liver cancer

EXPLANATION
The oldest known mycotoxins are aflatoxins produced by Aspergillus flavus and Aspergillus parasiticus fungi. Aflatoxins are teratogenic, mutagenic and carcinogenic compounds and represent one of the major etiological factors of liver cancer in Africa. Cereals, oilseeds, beans and dried fruits can be infected with aflatoxin-producing molds, most commonly the contamination is affecting peanut, therefore peanuts can only be marketed after aflatoxin control.

128
Q

PBH - 167
Food allergy:
A) symptoms are localized to the gastrointestinal tract
B) every fourth person is allergic to a food ingredient
C) usually means lactose sensitivity
D) an immune system-mediated pathological response to one (or sometimes multiple) component of food

A

ANSWER
D) an immune system-mediated pathological response to one (or sometimes multiple) component of food

EXPLANATION
The food allergy is an abnormal immune response to the food which is often IgE-mediated (Type I hypersensitivity) or delayed cell-mediated reaction (IV type .). Mostly, specific antibodies can be detected in the serum of patients. Food allergy symptoms can not only affect the gastrointestinal tract but could cause skin symptoms, asthma or anaphylactic shock. The prevalence among the adult population is about 2%, while in children its more frequent, approx.. 8%. Lactose sensitivity or intolerance is a non-allergic pathological state, caused by the decreased functioning of the lactase enzyme, due to which the main carbohydrate component of the cow’s milk, lactose can not be digested by the patients and this causes the gastrointestinal symtpoms.

129
Q

PBH - 168
Which is the most common food poisoning in Hungary nowadays?
A) Staphylococcus aureus intoxication
B) Salmonellosis
C) Bacillus cereus intoxication
D) Campylobacteriosis

A

ANSWER
D) Campylobacteriosis

EXPLANATION
Campylobacterosis has been the most frequently reported gastrointestinal disease in the EU Member States and in Hungary since 2005. In recent years, the number of reported salmonellosis cases is declining in many countries, including Hungary, although it is still one of the most common causes of gastrointestinal infections, similarly to Campylobacter infections, which involves the most commonly infants (0- 4 years old).

130
Q

PBH - 169
Hospitalization of the patient is essential if the cause of food poisoning is:
A) Staphylococcus
B) Salmonella
C) Campylobacter
D) Toxic mushroom

A

ANSWER
D) Toxic mushroom

EXPLANATION
Hospital admission is required if there is a suspicion of mushroom poisoning. In the other cases hospitalization is required only if the symptoms indicate it.

131
Q

PBH - 170
Which of the following definitions describes clinical effectiveness?
A) To achieve the greatest possible health gain compared to the available resources
B) Effective use of care resources at the given level
C) A higher level of health care than the care provided by family doctors and outpatient institutions
D) Minimizing the risk of side effects and complications

A

ANSWER
A) To achieve the greatest possible health gain compared to the available resources

EXPLANATION
The definition of clinical efficacy is to maximize health gain compared to available resources. This can only be achieved through the efficient use of resources and the minimization of complications, but these are the indicators of clinical effectiveness not the definition of clinical effectiveness itself.

132
Q

PBH - 171
Which primary prevention strategy involves the early risk/ risk factor detection/identification?
A) risk group strategy
B) Populational strategy
C) Both
D) None

A

ANSWER
A) risk group strategy

EXPLANATION
Population strategy of primary prevention includes preventive interventions for the whole population. There are no target groups here, information or interventions should reach everyone. The principle of the risk factor strategy is to identify individuals or groups with increased risk, and then aim these groups specifically with preventive services. Obviously, this strategy requires risk factor identification in order to select the target group properly

133
Q

PBH - 172
What phenomenon urged the /rethinking of the healthcare system of the developed countries in the second half of the 20th century?
A) The rapid growth of death rates related to chronic non-communicable diseases
B) Reduction in the payments related to health insurance due to increasing unemployment rate
C) the gap between GDP’s slow and the health care costs rapid growth
D) Significant development in the field of health informatics

A

ANSWER
C) the gap between GDP’s slow and the health care costs rapid growth

EXPLANATION
Mortality of chronic noncommunicable diseases did not grow intensively in the most industrialized countries in the second half of the 20th century, even a declining trend was observed related to cardiovascular mortality. The increasing trend in unemployment rate was not so intensive that it could affect the health care system itself. Informatics is not a crucial factor in the healthcare systems, on the contrary, it must adapt to the healthcare system as well as must serve its needs. The true reason was the very rapid growth of health care costs, which the GDP increase could not keep pace with. It was necessary to restructure the healthcare system, besides organizational changes rethinking its content was also necessary.

134
Q

PBH - 173
What is the most important factor of healthcare in the market-oriented countries?
A) accessibility
B) equality
C) quality
D) the above factors are not being investigated in market-oriented systems

A

ANSWER
C) quality

EXPLANATION
Market-oriented systems do not focus on democratic theorems but on the quality of service which comes closest to the mechanisms of the free market. The more restrictive regulation of market mechanisms is characteristic of socially sensitive systems which, however, are unable to provide a high quality level due to their limited resources.

135
Q

PBH - 174
Which country has the best balanced accessibility/equality/quality relation of the health care in the past three decades?
A) Hungary
B) USA
C) Canada
D) Poland

A

ANSWER
C) Canada

EXPLANATION
Formerly socialist countries have emphasized the accessibility and equality but were unable to sustain the proper quality (due to limited resources). The USA healthcare system was characterized by high quality, but this care was provided for narrower groups only. In Canada, the basic medical care is financed from taxes (a strong economic potential is available for this) and the additional treatments are financed mainly through additional insurances, so the Canadian healthcare system is considered as well organized.

136
Q

PBH - 175
The four-player model of healthcare services is extended with one of the following participants compared to the two-player everyday consumer model:
A) Provider
B) Politics (state)
C) Both
D) None

A

ANSWER
B) Politics (state)

EXPLANATION
Consumers and service providers are the two main participants of common consumer models. Obviusly these two participants are present in the field of health services as well. However, the health care consumer models are complemented with two other participants, namely with the financier (usually the health insurer, as the services are usually not obtained by the consumers directly) and wih the politics. Politics is the expression of state’s will related to healthcare.

137
Q

PBH - 176
In which case are the interests of the provider and the insurer contrary in the field of healthcare?
A) the amount of contribution paid by the insured person
B) The spectrum of services offered by the service provider facilites
C) Both
D) None

A

ANSWER
B) The spectrum of services offered by the service provider facilites

EXPLANATION
The healthcare provider is generally not interested in the question who is paying for its service. Furthermore, it not interested in the amount of self payment of the customer for receiving adequate service (of course, a too high self payment is not in the interest of the provider as it decreses the requisition of the service). Rather, the provider and the insurer have contrary interest in the spectrum of the services offered. Interest of the service provider is to offer as many types of service as possible (because it sells more and gets more reimbursed), while the insurer prefers a narrower range of the services because it has to pay less for less service.

138
Q

PBH - 178
How many active family doctors work in adult basic care in Hungary approximately?
A) 1000
B) 5000
C) 10000
D) 20000

A

ANSWER
B) 5000

EXPLANATION
Approximately 5,000 general practitioners (adult family doctors?) (and about 1,500 children family doctor) work in Hungary. Their number is almost identical to the number of practice districts, as the number of vacant practices in recent years has been around 150-200. In the near future, primary care will be severely challenged by the high rate of retirement of elderly GPs. The situation may be worsened further by the , increasing number of young doctors leaving the country (i.e. by the decrease of refill).

139
Q

PBH - 179
What is the most important determinant factor in the financing of general practices in Hungary?
A) the number of patients registered at the GP
B) Number of actual doctor-patient encounters
C) quality of the provided primarycare
D) Preventive services (and their quality) performed by the general practitioner

A

ANSWER
A) the number of patients registered at the GP

EXPLANATION
GP practices are financed by a per capita basis in Hungary, so the financing is based on the number of registered persons. Nevertheless, a degression can be found in financing if the registered patients number exceeds a certain limit.

140
Q

PBH - 180
What is the most common service type within the activity of the pulmonary care institutions in Hungary?

A) X-ray screening (“lung screening”)
B) complex screening
C) Other primary and secondary prevention services
D) Chronic care and treatment of pulmonologic patients

A

ANSWER
D) Chronic care and treatment of pulmonologic patients

EXPLANATION
Pulmonary care institutions are dealing manily with providing care for pulmonological patients. The volume of X-ray screening has been decreased as the method is not effective enough for detecting lung cancers, and, on the other hand, the prevalence of TBC does not justify the regalur screening at populational level. On the basis of its experience and infrastructure the pulmonary care network performs complex screening services to enhance prevention and to utilize the capacities, but this is not its main activity area..

141
Q

PBH - 182
Which of the following statements is not true related to blood provision in Hungary?
A) The tasks related to blood provision is managed by the Hungarian Red Cross
B) blood donation is voluntary
C) blood donation is free of charge
D) human blood can not be marketed

A

ANSWER
A) The tasks related to blood provision is managed by the Hungarian Red Cross

EXPLANATION
The main principles of blood provision are the following in Hungary. The tasks related to blood provision are managed by the Hungarian National Blood Transfusion Service (HNBTS). Hungarian Red Cross participates in the organization of blood donations. Blood donation is voluntary, unlike in some countries where a fee is payed for donatng blood. Human blood and blood products could not be marketed.

142
Q

PBH - 183
In which field do medical doctors work as entrepreneurs with the highest proportion?
A) Primary Care
B) Outpatient care - Dentists
C) Outpatient care - Specialists (except for dentists)
D) Active inpatient care

A

ANSWER
A) Primary Care

EXPLANATION
There is a relatively high proportion of physicians working in primary and outpatient care as enterpreneurs, while this proportion is below 10% in inpatient care and is mainly restricted to specific occupations and duties. Proportion of enterpreneurs working in outpatient care is between 50% and 70% (with a higher proportion in the case of dentists), while the majority of primary care physicians (around 85%) are entrepreneurs.

143
Q

PBH - 184
What are the main points of Bismarck’s health insurance system?
A) Health-related costs/expenditure are covered by taxes; the care is based on citizenship and belongs to everyone (civil right)
B) contribution is in relation to income, care is based on needs; the care is granted for those who are paying the contribuition
C) contribution and chosen service are defined in a contract; care is granted to the insured person
D) None of them

A

ANSWER
B) contribution is in relation to income, care is based on needs; the care is granted for those who are paying the contribuition

EXPLANATION
In Bismarck’s health insurance system, insured persons pay a contribution (in proportion related to their income) and therefore receive care according to their needs. The system is based on the principles of solidarity (for the insured), because the care does not depended on the amount of contribution paid but the entitlement is granted by the fact of paying contribution). The Beveridge system is characterized by the theorem that health care costs are financed by taxes and care is granted for everyone. In the case of risk insurances, the contribution payment is related to the risk and the services chosen.

144
Q

PBH - 185
How much time is spent on average by general practitioners with one patient?
A) 6-7 minutes
B) 10-12 minutes
C) 16-18 minutes
D) 25-27 minutes

A

ANSWER
A) 6-7 minutes

EXPLANATION
Based on Hungarian statistics, general practitioners serve 8-10 patients per hour in general, i.e. 6-7 minutes fall on one patient.

145
Q

PBH - 186
The main objectives of public health are:
1) continuous monitoring of the given population’s health status, identification of groups/individuals with increased risk, identification of health problems and public health priorities
2) development of health promotion and disease prevention programs
3) an investigation related to the availability of health services
4) establishing the healthcare framework

A) answers 1, 2 and 3 are correct
B) answers 1 and 3 are correct
C) answers 2 and 4 are correct
D) only answer 4 is correct
E) all of the answers are correct

A

ANSWER
A) answers 1, 2 and 3 are correct

EXPLANATION
Development of organizational framework of healthcare is not a public health objective; since in this field only supporting the planning is a task of the public health to ensure the general availability.

146
Q

PBH - 187
Findings leading to the establishment of modern epidemiology:
1) microbiological identification of pathogens
2) discovering the immune system
3) invention of antimicrobial chemotherapeutics
4) vaccination against smallpox

A) answers 1, 2 and 3 are correct
B) answers 1 and 3 are correct
C) answers 2 and 4 are correct
D) only answer 4 is correct
E) all 4 answers are correct

A

ANSWER
A) answers 1, 2 and 3 are correct

EXPLANATION
Development in the field of microbiology, immunology and pharmacology bring forth the qualitative changes (improvements) of “new age”/modern epidemiology. These findings include the identification of pathogens, discoveryof the body’s reactions against them and the discovery ofchemotherapeutics (antimicrobial drugs).

147
Q

PBH - 188
The definition of health defined by Stokes et al. combines the following:
1) anatomical integrity
2) ability to be “valuable” for the family, in the workplace and in society
3) ability to overcome physical, biological and social burdens/stress
4) the absence of polymorphism in the gene pool

A) answers 1, 2 and 3 are correct
B) answers 1 and 3 are correct
C) answers 2 and 4 are correct
D) only answer 4 is correct
E) all of the answers are correct

A

ANSWER
A) answers 1, 2 and 3 are correct

EXPLANATION
All components were mentioned in the definition created by Stokes et al., except the absence of polymorphism, which does not exist at individual level.

148
Q

PBH - 189
Which of the following public health activities is/are primary preventive intervention(s)?
1) cervical cancer screening
2) vaccination
3) breast tumour screening
4) programs related to the prevention of smoking addiction

A) answers 1, 2 and 3 are correct
B) answers 1 and 3 are correct
C) answers 2 and 4 are correct
D) only answer 4 is correct
E) all of the answers are correct

A

ANSWER
C) answers 2 and 4 are correct

EXPLANATION
Primary prevention’s aims are to maintain general health, to prevent health loss and diseases. Primary prevention aims at eliminating the potential disease sources (etiological and /or risk factors) completely or partially or forming and increasing the resistance against them. Screening programs for an early detection of the disease are considered as secondary preventive interventions.

149
Q

PBH - 190
Epidemiology:
1) uses population level methods
2) examines the distribution of health status/phenomena
3) examines the distribution of factors that affect health status
4) examines the distribution of changes in health status/phenomena

A) answers 1, 2 and 3 are correct
B) answers 1 and 3 are correct
C) answers 2 and 4 are correct
D) only answer 4 is correct
E) all of the answers are correct

A

ANSWER
E) all of the answers are correct

EXPLANATION
Epidemiology is a scientific field that observs the distribution and changes of those determinants that influence the health-related states or events in a given population in order to use its findings in the field of health promotion, disease prevention, medical care and rehabilitation.

150
Q

PBH - 191
Epidemiological studies refer to populations which
1) can be defined geographically
2) can be defined in time
3) members have one or more common characteristics
4) members do not have one or more charateristics

A) answers 1, 2 and 3 are correct
B) answers 1 and 3 are correct
C) answers 2 and 4 are correct
D) only answer 4 is correct
E) all of the answers are correct

A

ANSWER
E) all of the answers are correct

EXPLANATION
The study of health related states and determinants is always carried out in a given - well-defined - population. Such a population may be a group of individuals belonging together or merged on the basis of their unique/specific characteristics (eg pupils of a particular grade of a given school, workers or other employees of a particular factory or other the institution, people who have been treated with a certain surgical procedure). Alternatively, a population may be formed by the residents of a given administrative unit (eg the inhabitants of a country/region/county/settlement at a given time or interval.

151
Q

PBH - 192

The census
1) involves all members of the population
2) examines the population for a certain time
3) in Hungary census was first conducted between 1784 and 1787
4) examines a representative sample of the population

A) answers 1, 2 and 3 are correct
B) answers 1 and 3 are correct
C) answers 2 and 4 are correct
D) only answer 4 is correct
E) all 4 answers are correct

A

ANSWER
A) answers 1, 2 and 3 are correct

EXPLANATION
The basic data of structural demography is provided by the census that counts all individuals on the basis of well-defined structural characteristics for a given time. Censuses are usually conducted in every 10 years in developed countries. Investigations of the ongoing changes between two censuses are based on the data obtained from calculations using extra- and interpolational methods. The ongoing changes can also be examined by analysises conducted on representative samples (1-5% of the population; microcensuses). The traces of censuses can be detected in the ancient times. Modern censuses are conducted since the 17th-18th century. The first census was implemented in Hungary due to the will of József the 2nd between 1784 and 1787 while the next census was. only in conducted in 1869. Since then, censuses are usually conducted in every 10 years.

152
Q

PBH - 193
Ecological studies:
1) usually produce correlation indicators
2) create and analyse scatter diagrams
3) are based on data collection at population level
4) evaluate relations of parameters of various groups

A) answers 1, 2 and 3 are correct
B) answers 1 and 3 are correct
C) answers 2 and 4 are correct
D) only answer 4 is correct
E) all of the answers are correct

A

ANSWER
E) all of the answers are correct

EXPLANATION
Ecological studies are a specific type of descriptive epidemiological studies which examine population level connections of characeristics on the basis on aggregated data by evaluating correlation indicators. Consequently, ecological studies are also called correlational studies. The two names emphasize different features of the same method. The term “ecological study” indicates that the examination is based not on individual data but on the average parameters of populations (or their subgroups). The term “correlation study” indicates that the direction and the strength of the correlation is described by correlation coefficients or regression equations. Results of the ecological studies are usually presented on scatter diagrams.

153
Q

PBH - 194
Analytical epidemiological studies
1) most common types are the case/control and the cohort studies
2) Testhypothetical relationships using aggregated or individual data
3) evaluate the role of significance of etiological factors
4) Analyze data obtained from observation or generated by experimental procedures.

A) Answers 1, 2 and 3 are correct
B) answers 1 and 3 are correct
C) Answers 2 and 4 are correct
D) only answer 4 is correct
E) all 4 answers are correct

A

ANSWER
B) answers 1 and 3 are correct

EXPLANATION
Analytical studies mainly focus on “why” type questions: they examine and test hypothetical relationships, identify the influencing (risk or preventive) factors, measure the role (significance) of the risk factors in the development of the phenomenon (risk). The two basic types of analytical epidemiological studies are the case/control and cohort studies, which primarily aim is to elucidate the etiological background of the diseases (occurring events)

154
Q

PBH - 197
Which health monitoring method (s) can be used when the information required to operate the monitor goes beyond the routine information requirements and which is not required by law and requires additional work and commitment of the data providers?
1) Polls
2) Linking custom data from other databases
3) Disease (group)-specific registries
4) Voluntary data providers

A) Answers 1, 2 and 3 are correct
B) Answers 1 and 3 are correct
C) Answers 2 and 4 are correct
D) only 4 is right
E) all 4 answers are correct

A

ANSWER
D) only 4 is right

EXPLANATION
Based on the information needs and data sources the following types of surveillance can be defined: 1. Mandatory reporting system (E.g. infectious diseases, occupational diseases).They are needed when the report requires some sort of intervention. Consequently, the content of the report is defined in detail according to the conditions of the intervention. 2. Monitoring based on laboratory results. (E.g.: Monitoring of antibiotic resistance) By regularly evaluating the results of microbiological laboratories, changes can be monitored relatively simply and effectively even at institutional level.3. Disease (group) specific registry (E.g .: tumor register, register of developmental malformations) Tools that register every diesease of a certain population, which use exact definition for the cases, collects detailed informations and often enable follow up of patienst. These tools require relatively large amounts of resources and rarely contain information regarding determinants of health. 4. Voluntary reporting (E.g.: Family doctor monitoring systems in regard to certain illnesses ,prevalency and quality of care; the General Practitioners Sentinal Station Programme operated by the Faculty of Public Health of the University of Debrecen). Ocassionally for operating the monitoring system such information is needed that goes beyond the routine information requirements and which is not required by law and requires additional work and commitment of the data providers.. In these cases it is advisable to generate a longer list from voluntary data providers, from whom a representative survey group can be set up. 5. Surveys (eg: National Health Interview Survey in 2000 and 2003, the European Health Interview Survey, 2009) These can be interview based, where the knowledge related to health, attitude and actions towards health as well as and experiences with the healthcare system and the health status based on self-assessment can be evaluated. It may require more resources, but the final data will be more reliable if this is combined with objective measurements as well. 6, Utilising databases of other information systems. (E.g.: Database of the National Health Insurance Fund, IT databases of healthcare providers.) These systems are primarily not for monitoring morbidity; the data content is generally insufficient for the monitoring, their data quality is often insufficient, its reliability varies from one provider to another, as institutional diagnostic and coding methods vary. We can get misleading results if we use these databases without taking into account these limiting factors. 7. Monitoring Warning Events (eg diagnosis of Mesothelioma) This method can be used in the case of rare diseases that are connected to a risk factor in a very specific way and are automatically linked to a certain intervention..8. Combining custom data from databases (individual IDs can be used to connect different databases. In a pooled database of linked exposure and health data, little effort is required to detect specific connections. It is a very potent method in regard to data utilization, however this possibility is very limited in certain countries due to legisislation. It is also useful to determine the reporting discipline of some registries.) 9. Combined Monitor (Eg .: Northeastern Hungarian Health Observatory) We can create a more complete picture of a disease by evaluating the results of multiple monitors together, for example if combined evaluation of mortality, morbidity and the quality of care data is possible.

155
Q

PBH - 198
An indicator is considered as well defined when
1) its calculation is based on a standardized method.
2) it is connected to interventions.
3) its results can be compared in different populations.
4) it is easy to interpret.

A) answers 1, 2 and 3 are correct
B) answers 1 and 3 are correct
C) answers 2 and 4 are correct
D) only answer 4 is correct
E) all of the answers are correct

A

ANSWER
E) all of the answers are correct

EXPLANATION
An indicator can be considered as well defined, when it can be easly interpreted, it is connected to intervention possibilites, correlates with other indicators, is timely, can be produced in all study populations, its raw data originate from reliable data sources, does not need too many resources, its calculation is based on standardized methodology, it reflects reality in a valid manner, its values are comparable in different populations, and is related to a real public health issue.

156
Q

PBH - 199
The following quality criteria can be specified for user protection at healthcare websites:
1) the website should contain the exact name, postal and electronic address of the service provider
2) authors of the website should be easily identified
3) the last update must be indicated clearly
4) If the site recommends the use of medications, then it must indicate that the recommendation does not replace the personal advice of professionals.

A) Answers 1, 2 and 3 are correct
B) Answers 1 and 3 are correct
C) Answers 2 and 4 are correct
D) only answer 4 is correct
E) all 4 answers are correct

A

ANSWER
E) all 4 answers are correct

EXPLANATION
For healthcare websites, the following quality criterias can be formulated regarding user protection: 1. Transparency (It is generally expected that the website must contain the exact name, the postal and electronic address of the service provider, the target group of the publication , the aim of the site and a listing of the a participants in the development of the webpage, as well as its supporters.) 2. Authenticity of the data source (The website must display the data that enables user to identify the authors of the site,it must also display the sources of the published information together with the date of publication. If the use of any methods, medicine or devices is recommended by the website, then it must be indicated that the recommendation does not replace the personal advice of professionals. Date of the last update must be also shown on the website.) 3. Responsibility (It must provide options for user feedback, it also has to indicate who will be responsible for answering these user inputs.).

157
Q

PBH - 200
How does the treatment threshold depend on the benefit and risk of the treatment?
1) treatment threshold = 1 / [risk / benefit +1]
2) treatment threshold = 1 / [benefit / risk +1]
3) treatment threshold = benefit / (risk + benefit)
4) treatment threshold = risk / (benefit + risk)

A) Answers 1, 2 and 3 are correct
B) Answers 1 and 3 are correct
C) answers 2 and 4 are correct
D) only answer 4 is correct
E) all 4 answers are correct

A

ANSWER
C) answers 2 and 4 are correct

EXPLANATION
The relationship between treatment threshold and benefit: treatment threshold = risk /( benefit + risk) = 1 / [benefit / risk +1]

158
Q

PBH - 202
Average life expectancy at birth in Hungary between 1988 and 1993
1) for men, it has been gradually dropping behind the Austrian reference level
2) For women, it has been increasing steadily
3) For women, it has been gradually dropping behind the Austrian reference level
4) for men, it has been decreasing steadily

A) statements 1, 2 and 3 are corrrect
B) statements 1 and 3 are correct
C) statements 2 and 4 are correct
D) only statement 4 is correct
E) all statement are correct

A

ANSWER
E) all statement are correct

EXPLANATION
As for men, the average life expectancy at birth calculated on the basis of annual mortality indicators decreased slightly between 1988-1993 - which was a unique phenomenon in Europe. On the contrary, slightly but consequently increasing trend has been observable in case of women since 1980. However, in both genders this indicator is far behind the same indicators of not only the most developed countries, but also of Czechia and Poland, the previous sister states showing similar historical fate. When comparing the Hungarian indicator regarding the average life expectancy to that of the male and female population of Austria (a country that has similar geografical location and - apart from the second half of the 20th century similar history) the lagging behind was showing a constantly increasing trend from 1980 to the millenium and has been stable since that. This lag was 7.9 years in case of men and 5.1 years in case of women in 2008.

159
Q

PBH - 203
The revisions of the International Classification of Diseases (ICD) are characterized by the followings:
1) Revision 10 is currently valid
2) Revision 11 is currently in progress
3) professional reviews are supervised by the WHO
4) Class boundaries will be ereased in the 11th revision

A) answers 1, 2 and 3 are correct
B) Answer 1 and 3 are correct
C) Answer 2 and 4 are correct
D) only answer 4 is correct
E) all 4 answers are correct

A

ANSWER
A) answers 1, 2 and 3 are correct

EXPLANATION
The tenth, currently valid revision of the ICD was implemented on 1 Jan 1996. The 11th revision of ICD is currently in progress and the expected year of its completion is 2014. The revisions have been supervised by the WHO since 1948. One of the most important elements of revision is to specify better the class boundaries rather than to erase them.

160
Q

PBH - 204
The incidence of new malignant neoplasms is characterized by the following in Hungary
1) The most frequent malignancy in men is prostate cancer
2) The most frequent malignancy in men is bronchial and lung cancer
3) The most frequent malignancy in women is bronchial and lung cancer
4) The most frequent malignancy in women is breast cancer

A) Answers 1, 2 and 3 are correct
B) Answers 1 and 3 are correct
C) Answers 2 and 4 are right
D) only answer 4 is correct
E) all 4 answers are correct

A

ANSWER
C) Answers 2 and 4 are right

EXPLANATION
Based on National Registry on Cancer data, the following malignancies were the most frequently diagnosed in Hungary in 2009: for men, bronchial and lung cancer (19.5%), cancer of the the lips, the oral cavity and the pharynx (10.5%) , colorectal and anal cancer (10%). In case of women, breast cancer (21.5%), bronchial and lung cancer (12%), colorectal and anal cancer (8%), and cervical cancer (4%). The percentage distribution indicates the order in both sexes.

161
Q

PBH - 205
Amongst the etiological factors of malignancies, in case of men the size of which risk factor(s) reaches or exceeds 10% ?
1) lifestyle factors
2) smoking
3) exposure to sunlight
4) occupational exposure

A) answers 1, 2 and 3 are correct
B) answers 1 and 3 are correct
C) answers 2 and 4 are correct
D) only answer 4 is correct
E) all of the answers are correct

A

ANSWER
A) answers 1, 2 and 3 are correct

EXPLANATION
Sizes of risk factors among the aetiological factors for men: lifestyle factors 35%, smoking 30%, sunlight 10%, occupational exposure 5%, alcohol 5%.

162
Q

PBH - 206
Carcinogenic nutritional ingredients / factors:
1) N-nitroso compounds
2) flavonoids
3) low-fibre diet
4) beta-carotene

A) answers 1, 2 and 3 are correct
B) answers 1 and 3 are correct
C) answers 2 and 4 are correct
D) only answer 4 is correct
E) all of the answers are correct

A

ANSWER
B) answers 1 and 3 are correct

EXPLANATION
N-nitroso compounds, which are formed in the intestinal tract from nitrites and nitrates, may be involved in the development of esophageal, gastrointestinal, bowel and bladder cancers. Low-fibre diet is a factor that increases the risk of developing colon cancer. Flavonoids and beta-carotene have a protective effect.

163
Q

PBH - 207
Breast screening is recommended in Hungary as follows:
1) for women aged 50-65
2) for women aged 45-65
3) annually
4) biannually

A) Answers 1, 2 and 3 are correct
B) Answers 1 and 3 are correct
C) Answers 2 and 4 are right
D) only answer 4 is correct
E) all 4 answers are correct

A

ANSWER
C) Answers 2 and 4 are right

EXPLANATION
Breast cancer screening (mammography) is performed biannually for the age group 45-65 in Hungary

164
Q

PBH - 208
Epidemiological studies proving the relationship between being overweight/obese and the onset of coronary heart disease:
1) North Karelia project
2) Pritkin-Ornish
3) Framingham Heart Study
4) National Cholesterol Education Committee

A) Answers 1, 2 and 3 are correct
B) Answers 1 and 3 are correct
C) Answers 2 and 4 are correct
D) only answer 4 is correct
E) all 4 answers are correct

A

ANSWER
B) Answers 1 and 3 are correct

EXPLANATION
The relationship between being overweight/obese and the onset of coronary heart disease was confirmed by the North Karelia project and the Framingham Heart study. The National Cholesterol Education Committee deals with the risk-increasing effects of cholesterol in the aspect of causing coronary heart disease, whereas the name of the most restricted fat intake reducing diet is Pritkin-Ornish.

165
Q

PBH - 209
According to the recommendation based on the cooperation of the World Health Organization and the American Society of Diabetes, diabetes screening is characterized by the following:
1) it is reasonable above the age of 45
2) it is based on determining fasting blood sugar levels
3) In case of normal test results, the repetition of the test on a three yearly basis seems to be appropiate
4) it is based on the oral glucose tolerance test

A) answers 1, 2 and 3 are correct
B) Answer 1 and 3 are correct
C) Answer 2 and 4 are correct
D) only answer 4 is correct
E) all 4 answers are correct

A

ANSWER
A) answers 1, 2 and 3 are correct

EXPLANATION
Earlier start and more freuqent screening is suggested for individuals whose BMI reaches or exceeds 27 kg/m2, who has straight-line relatives with diabetes, who have offspring(s) with a birth weight over 4 kg, who have have 2.9 mmol or higher triglyceride levels and/or have decreased HDL cholesterol level (0.9 mmol/l or lower). According to the recommendation regular diabetes screening based on assesing fasting glucose is suggested for persons above the age of 45. In case of a normal test result, repetition of the test on a three yearly basis seems to be appropiate

166
Q

PBH - 210
Risk factor(s) of osteoporosis is (are):
1) decrease in oestrogen production
2) physical inactivity
3) smoking
4) D-hypervitaminosis

A) Answers 1, 2 and 3 are correct
B) Answers 1 and 3 are correct
C) Answers 2 and 4 are correct
D) Only answer 4 is correct
E) All of the answers are correct

A

ANSWER
A) Answers 1, 2 and 3 are correct

EXPLANATION
The main risk factors of osteoporosis are: decrease in estrogen production, physical inactivity and smoking. D-hypervitamonis causes bone alterations characterized by increased calcification.

167
Q

PBH - 211
Liver diseases and cirrhosis in Hungary
1) accounts for approximately 80% of premature death cases caused by gastrointestinal diseases
2) about 80% of these diseases has an alcoholi-related origin
3) their high prevalence can be explained by the amount of alcohol consumed
4) their high prevalence can be explained by the quality of alcohol consumed

A) answers 1, 2 and 3 are correct
B) answers 1 and 3 are correct
C) answers 2 and 4 are correct
D) only answer 4 is correct
E) all of the answers are correct

A

ANSWER
E) all of the answers are correct

EXPLANATION
Diseases of the liver and cirrhosis are responsible for 80% of premature death caused by gastrointestinal disorders in Hungary. Approximately 80% of these diseases has an alcohol-related origin, their high prevalence can be explained by the amount and quality of alcohol consumed (consumption of alcoholic beverages contaminated by aliphatic alcohols and obtained from illegal sources)

168
Q

PBH - 215
Strategy for the prevention of accidental injuries and accidental deaths:
1) its aim is the accident control instead of accident prevention
2) the best known strategy is Haddon’s strategy
3) is mainly not related to public health
4) focuses on the safe designing of vehicles and trafic areas

A) answers 1, 2 and 3 are correct
B) answers 1 and 3 are correct
C) answers 2 and 4 are correct
D) only answer 4 is correct
E) all 4 answers are correct

A

ANSWER
E) all 4 answers are correct

EXPLANATION
The strategy of preventing accidental injuries and accidental deaths is basically not a designed public health field. Haddon’s technical accident control strategy is consisted of ten, logically coherent strategic steps, which describe safe designing of the vehicles, traffic areas, etc. Literature is using the term injury controll mostly instead of using the term accident prevetnion (eg the safety belt is for accident prevention, but it can also cause injuries)

169
Q

PBH - 216
Strategic groups of suicide prevention are the followings:
1) Training of professionals who are able to identify individuals with increased risk
2) Elimination of the identified risk factors as much as possible
3) Providing direct help through the use of crisis ambulances and helplines
4) Employment program aiming at the mentally ill people

A) answers 1, 2 and 3 are correct
B) Answer 1 and 3 are correct
C) Answer 2 and 4 are correct
D) only answer 4 is correct
E) all 4 answers are correct

A

ANSWER
A) answers 1, 2 and 3 are correct

EXPLANATION
The literature distinguishes five strategic groups of interventions related to suicide prevention. These include all methods except the employmet of persons with mental disorders.

170
Q

PBH - 217
The IPV is characterized by the followings:
1) it contains inactivated viruses
2) it is an oral vaccine
3) the last vaccine is being given to children of 6 years old in Hungary
4) the last vaccine is being given to children of 15 months old in Hungary

A) Answers 1, 2 and 3 are correct
B) answers 1 and 3 are correct
C) Answer 2 and 4 are correct
D) only answer 4 is correct
E) all 4 answers are correct

A

ANSWER
B) answers 1 and 3 are correct

EXPLANATION
IPV is a vaccine containing inactivated polyoviruses which should be given intramuscularly. The administration of the vaccine (containing also other components) takes place in age-related manner according to the vaccination regulation is performed that is effective in Hungary since 1 January 2010. The vaccine contains other components as well. The vaccine containing the components of DTPa (diphtheria, tetanus, acellularis pertussis), IPV (inactivated polyovirus vaccine) and Hib (haemophilus influenzae b) together must be given to childrem of 2, 3, 4 and 18 months afe, while DTpa and IPV vaccination must be given at the age of 6 years (this is the last IPV vaccination).

171
Q

PBH - 220
If botulism infection is suspected, then the following samples should be sent to the laboratory:
1) feces
2) food leftovers
3) vomit
4) blood

A) answers 1, 2 and 3 are correct
B) answers 1 and 3 are correct
C) answers 2 and 4 are correct
D) only answer 4 is correct
E) all 4 answers are correct

A

ANSWER
C) answers 2 and 4 are correct

EXPLANATION
If botulism infection is suspected, samples of food leftovers and blood must be sent to the laboratory. The bacteria and toxins can be detected from food leftovers while the toxin absorbed and entered the circulation can be detected from blood samples. Sending feces samples does not make sense due to the fact that the symptoms are caused by swallowed and absorbed toxins.

172
Q

PBH - 222
Final disinfection is mandatorial in the following infectious diseases:
1) Cholera
2) Infectious hepatitis
3) Diphtheria
4) Salmonellosis

A) answers 1, 2 and 3 are correct
B) answers 1 and 3 are correct
C) answers 2 and 4 are correct
D) only answer 4 is correct
E) all of the answers are correct

A

ANSWER
E) all of the answers are correct

EXPLANATION
The listed diseases may be transmitted by the faeco-oral route and by body fluids, consequently, continuous and final disinfection is needed in order to prevent the infection from spreading. Final disinfection with more strict regulations has to be carried out in case of cholera infection.

173
Q

PBH - 224
The following reported infectious diseases should be unreported:
1) in case of long lasting changes affecting any organ
2) in case of complications
3) in case of lethal outcome
4) in case of severe diseases

A) answers 1, 2 and 3 are correct
B) Answer 1 and 3 are correct
C) Answer 2 and 4 are correct
D) only answer 4 is correct
E) all 4 answers are correct

A

ANSWER
A) answers 1, 2 and 3 are correct

EXPLANATION
Unreporting must be done in case of long lasting changes effecting any organ, complications or fatal outcome.

174
Q

PBH - 225
In which disease(s) should the patient be separated/isolated/quarantined at the joint Szent István - Szent László Hospital?
1) Hepatitis A
2) AIDS
3) typhoid abdominalis
4) acute anterior polyomyelitis

A) Answers 1, 2 and 3 are correct
B) Answer 1 and 3 are correct
C) Answer 2 and 4 are correct
D) only 4 is right
E) all 4 answers are correct

A

ANSWER
D) only 4 is right

175
Q

PBH - 226
Clearance investigations are required in case of infection with the following pathogen(s):
1) Salmonella enteritidis
2) Salmonella typhi
3) Vibrio cholerae
4) Yersinia enterocolitica

A) answers 1, 2 and 3 are correct
B) answers 1 and 3 are correct
C) answers 2 and 4 are correct
D) only answer 4 is correct
E) all 4 answers are correct

A

ANSWER
A) answers 1, 2 and 3 are correct

EXPLANATION
Clearing examinations are needed in the following infections: 1. Salmonellosis. After recovering, a fecal bacteriological control (release) test should be carried out only in persons who attend communities of children of 0 to 3 years, who are receiving care in social or health care institutions and who are working with breast milk or are giving breast milk. These persons are allowed to continue their activity in case of having two normal stool tests on two consecutive days started two days after the clinical recovery. 2. Typhus abdominalis. The patient should be isolated until tests conducted on fecal and urine samples on 3 consecutive days starting 48 hours after the cease of the clinical symptoms and the end of the antibiotic therapy give negative results. If only one of the tests gives positive result then the isolation may be discontinued, but the patient should undergo a clinical observation until the pathogen shedding/carrying is ceased. 3. Cholera. The patient cured from cholera have to be isoleted until bacteriological stool tests on two consecutive days starting 48 hours after clinical recovery and end of the antibiotic therapy give negative result. 4. After the recovery of infection with Yersiniosis no mandatory release tests are required.

176
Q

PBH - 227
ETEC group of E. coli is characterized by the followings:
1) causes dysenteria in infants
2) causes cholera-like symptoms in adults
3) causes dysenteria-like symptoms in adults
4) causes asymptomatic excretion/shedding

A) answers 1, 2 and 3 are correct
B) answers 1 and 3 are correct
C) answers 2 and 4 are correct
D) only answer 4 is correct
E) all of the answers are correct

A

ANSWER
C) answers 2 and 4 are correct

EXPLANATION
ETEC = Enterotoxic (or enterotoxigenic) E. coli. Its toxins paralyze the Na + -K + pump in the epithelial cells of the intestinal wall, so the infection cause profuse cholera-like diarrhoea. As with most coliform infections, asymptomatic shedding is also possible. Coli infections may also occur in the form of EPEC (enteropathogenic E. coli, intestinal mucosal degeneration) and EIEC (enteroinvasive E. coli, dysenteria-like symptoms, mainly attacking the colon) in Hungary. Beyond the above forms, enteroadhesive, enterohaemorrhagic and enteroaggregative E. coli groups are also known but these forms appear in Hungary never or on very rare occasions.

177
Q

PBH - 229
Select which of the listed diseases are mandatory to be screened for in certain work-related activities (in child-care, educational and health institutions, food industry, etc.).
1) syphilis
2) gonorrhea
3) HIV
4) genitourinary chlamydial infection

A) answers 1, 2 and 3 are correct
B) answers 1 and 3 are correct
C) answers 2 and 4 are correct
D) only answer 4 is correct
E) all 4 answers are correct

A

ANSWER
A) answers 1, 2 and 3 are correct

EXPLANATION
As specified in various acts, screening tests should be performed in certain jobs in order to detect syphilis, gonorrhea and HIV infection to prevent the transmissions of these diseases.

178
Q

PBH - 230
In which secretion of the patient can the Lyssavirus be detected?
1) saliva
2) tears
3) urine
4) milk

A) answers 1, 2 and 3 are correct
B) answers 1 and 3 are correct
C) answers 2 and 4 are correct
D) only answer 4 is correct
E) all of the answers are correct

A

ANSWER
E) all of the answers are correct

EXPLANATION
After the virus has entered the central nervous system, it begins to grow intensely and then travels along the efferent nerves to all organs. As a result of this, it will appear in all tissues and - listed - excretions.

179
Q

PBH - 231
Active immunization is used to prevent the following diseases:
1) meningitis epidemica
2) scarlet fever
3) pneumococcal meningitis
4) legionellosis

A) answers 1, 2 and 3 are correct
B) answers 1 and 3 are correct
C) answers 2 and 4 are correct
D) only answer 4 is correct
E) all 4 answers are correct

A

ANSWER
B) answers 1 and 3 are correct

EXPLANATION
Active immunization is used against meningitis epidemica, which is mandatory in certain occupations (eg laboratory technicians), otherwise it is recommended, especially for those who are more susceptible. Active immunization is available against pneumococcus meningitis (free of charge for those younger than 2 years old) which is highly recommended for those with increased risk.There is no active immunization against scarlet fever and legionellosis.

180
Q

PBH - 232
Which of the listed pathogens play(s) a role in the development of tumors?
1) HPV
2) HBV
3) HCV
4) HAV

A) answers 1, 2 and 3 are correct
B) answers 1 and 3 are correct
C) answers 2 and 4 are correct
D) only answer 4 is correct
E) all of the answers are correct

A

ANSWER
A) answers 1, 2 and 3 are correct

EXPLANATION
Among the listed pathogens, HPV participates in the development of cervical cancer, HBV and HCV could cause liver cancers; while HAV is not a carcinogenic virus.

181
Q

PBH - 233
Patients infected with the following disease(s) should be isolated:
1) meningitis epidemica
2) parotitis epidemica
3) scarlet fever
4) legionellosis

A) answers 1, 2 and 3 are correct
B) answers 1 and 3 are correct
C) answers 2 and 4 are correct
D) only answer 4 is correct
E) all of the answers are correct

A

ANSWER
A) answers 1, 2 and 3 are correct

EXPLANATION
Patients infected with meningitis epidemica or parotis epidemica must be isolated in rooms for infectious diseases. Patients infected with scarlat fever must be isolated at their own home or at rooms for ifectious diseases. The reason behind the isolation is that all three types of diseases are transmitted by humans. Since the transmission of the two forms of Legionellosis from humans to humans does not play a role, the isolation is not necessary in these cases.

182
Q

PBH - 234
In which case(s) should chemoprophylaxis be used on the contact persons?
1) Diphtheria
2) Meningitis epidemica
3) H. influenzae meningitis
4) Pertussis

A) answers 1, 2 and 3 are correct
B) answers 1 and 3 are correct
C) answers 2 and 4 are correct
D) only answer 4 is correct
E) all of the answers are correct

A

ANSWER
E) all of the answers are correct

EXPLANATION
People who were in direct contact with the patient infected with any of the four listed diseases should receive antibiotic prophylaxis.

183
Q

PBH - 235
Admission to the given health care facility should be suspended if the disease(s) below take(s) place in the institution:
1) scarlet fever
2) meningitis caused by H. influenzae
3) invasive disease caused by Pneumococcus pneumoniae
4) meningitis epidemica

A) answers 1, 2 and 3 are correct
B) answers 1 and 3 are correct
C) answers 2 and 4 are correct
D) only answer 4 is correct
E) all of the answers are correct

A

ANSWER
D) only answer 4 is correct

EXPLANATION
Blocking the hospital admissions in the case of meningitis epidemica is necessary, because the probability of transmission of the pathogen is 50. For this reason, persons entering the area (hospital ward) are exposed to an increased risk of being infected. Moreover, admisson blocking is justified by the high lethatility of the disease which is 70% without antibiotic therapy, 15-20% with antibiotic therapy. For the other three diseases admission blocking is not necessary.

184
Q

PBH - 239
Which disease(s) is (are) spread by cats?
1) Q fever
2) Lyssa (rabies)
3) Tularemia
4) Toxoplasmosis

A) answers 1, 2 and 3 are correct
B) answers 1 and 3 are correct
C) answers 2 and 4 are correct
D) only answer 4 is correct
E) all of the answers are correct

A

ANSWER
E) all of the answers are correct

EXPLANATION
In all listed diseases cats may be the sources of infection although the frequency may vary. The role of cats is less important in the transmisson of Q fever and tularemia than the role of other animals. Cats belong, however, to the most important sources of Rabies; the most recent lethal Rabies cases were due to cats because of the late diagnosis of the infections The role of the cats in the transmission of toxoplasmosis should be especially emphasized. Toxoplasmosis is a teratogenic infection. Since 10-15% of cats - and not only those living free but also the domestic ones - are constantly infected with Toxoplasma and shed the pathogens with their feces, pregnant women should avoid direct contact with them.

185
Q

PBH - 240
Which of the listed wastes can not be destroyed by incineration?
1) radioactive waste
2) waste containing more than 40% chlorine
3) which causes the formation of carcinogenic substances during incineration
4) which have heavy metal content of more than a few percent

A) answers 1, 2 and 3 are correct
B) answers 1 and 3 are correct
C) answers 2 and 4 are correct
D) only answer 4 is correct
E) all of the answers are correct

A

ANSWER
E) all of the answers are correct

EXPLANATION
All waste types listed - radioactive waste, waste containing more than 40% chlorine, those which cause the formation of carcinogenic substances during their burning, which have a heavy metal content of more than a few percent are not permitted to deal with in waste incinerators. These wastes must be preserved/settled in special waste cemeteries.

186
Q

PBH - 241
Which of the listed substances could cause hematopoietic cancer in humans due to their chronic exposure?
1) vinyl chloride
2) benzidine
3) nitrosamine
4) benzene

A) answers 1, 2 and 3 are correct
B) answers 1 and 3 are correct
C) answers 2 and 4 are correct
D) only answer 4 is correct
E) all of the answers are correct

A

ANSWER
D) only answer 4 is correct

EXPLANATION
Chronic effects of benzene could cause hematopoietic tumors, leukemia. Vinyl chloride could cause hemangiosarcomas, benzidine could cause bladder cancer, while nitrosamine exposure could cause stomach cancer.

187
Q

PBH - 242
Which gas(es) can cause pulmonary oedema when inhaled at high concentration?
1) ethylene
2) phosgene
3) hydrogen sulfide
4) fluorine

A) answers 1, 2 and 3 are correct
B) answers 1 and 3 are correct
C) answers 2 and 4 are correct
D) only answer 4 is correct
E) all 4 answers are correct

A

ANSWER
C) answers 2 and 4 are correct

EXPLANATION
High concentrations of phosgene and fluorine could cause irritation in the respiratory tract during their acute toxication and eventually could cause lung oedema. Ethylene as asphyxiant gas at high concentration in the air, could cause hypoxia which could result in suffocation due anoxia. Hydrogen sulfide - although it irritates the airway mucosa - could cause hyperpnoe, spasms, loss of consciousness and eventually coma.

188
Q

PBH - 243
Cadmium of the soil is accumulated in the following plant(s):
1) tobacco plant
2) leafy vegetables
3) potato
4) cereals

A) answers 1, 2 and 3 are correct
B) answers 1 and 3 are correct
C) answers 2 and 4 are correct
D) only answer 4 is correct
E) all 4 answers are correct

A

ANSWER
E) all 4 answers are correct

EXPLANATION
All listed plants are able to accumulate cadmium absorbed from the soil, their consuming (smoking, eating) leads to chronic metal exposition. This represents a particular danger, especially when the Cd content level is high in the soil mainly due to industrial activity.

189
Q

PBH - 244
Which of the following wastes is/are considered as hazardous waste(s)?
1) manure of livestock farms
2) unused pesticides
3) medical waste
4) household chemical products

A) Answers 1, 2 and 3 are correct
B) Answers 1 and 3 are correct
C) Answers 2 and 4 are correct
D) only answer 4 is correct
E) all 4 answers are correct

A

ANSWER
E) all 4 answers are correct

EXPLANATION
All four types of waste listed - such as livestock manure, pesticides that are not used for plant protection, wastes generated by health care activities and household chemical products - are considered as hazardous wastes due to the infectivity and / or toxicological effects.

190
Q

PBH - 245
Which muscarinic symptoms can be alleviated by the addition of atropine in organic phosphate ester poisoning?
1) bronchospasm
2) bradycardia
3) narrowed pupils (pinpoint pupils)
4) spasms of the respiratory muscles

A) answers 1, 2 and 3 are correct
B) answers 1 and 3 are correct
C) answers 2 and 4 are correct
D) only answer 4 is correct
E) all of the answers are correct

A

ANSWER
A) answers 1, 2 and 3 are correct

EXPLANATION
The first three symptoms (bronchospasm, bradycardia, pinpoint pupils) are muscarine-like symptoms, therefore they can be mitigated by atropine, while the fourth symptom, the spasm of the respiratory muscles, as being a nicotinic answer, could not be treated with atropine.

191
Q

PBH - 246
Which of the following four heavy metals has/have human mutagenic effect?
1) cadmium
2) cobalt
3) chrome
4) lead

A) answers 1, 2 and 3 are correct
B) answers 1 and 3 are correct
C) answers 2 and 4 are correct
D) only answer 4 is correct
E) all of the answers are correct

A

ANSWER
B) answers 1 and 3 are correct

EXPLANATION
Cadmium and chromium have mutagenic effect in humans, while cobalt and lead do not have any mutagenic effect in humans.

192
Q

PBH - 247
Pregnant women are not allowed to be employed at a given workplace if it means exposition to the following metal(s):
1) Nickel
2) Lead
3) Mercury
4) Arsenic

A) answers 1, 2 and 3 are correct
B) answers 1 and 3 are correct
C) answers 2 and 4 are correct
D) only answer 4 is correct
E) all 4 answers are correct

A

ANSWER
E) all 4 answers are correct

EXPLANATION
Employing pregnant women exposed to any metal listed is contraindicated

193
Q

PBH - 250
Which are the characteristics of London-type smog?
1) It is reducing type smog.
2) Nitrogen oxide plays a role in its mechanism.
3) Sulfur dioxide plays a role in its mechanism.
4) UV radiation is required to its formation.

A) answers 1, 2 and 3 are correct
B) answers 1 and 3 are correct
C) answers 2 and 4 are correct
D) only answer 4 is correct
E) all 4 answers are correct

A

ANSWER
B) answers 1 and 3 are correct

EXPLANATION
The London-type smog is considered as reducing smog, the following chemical components play roles in its mechanism of action: carbon dioxide, carbon monoxide, sulfur dioxide. Nitrogen oxide and UV radiation are not involved in thise mechanism, these component are the preconditions of the Los Angeles-type oxidative smog.

194
Q

PBH - 254
Complication(s) of silicosis is/are the followings:
1) Raynaud syndrome
2) tuberculosis
3) pleural mesothelioma
4) cor pulmonale

A) answers 1, 2 and 3 are correct
B) answers 1 and 3 are correct
C) answers 2 and 4 are correct
D) only answer 4 is correct
E) all of the answers are correct

A

ANSWER
C) answers 2 and 4 are correct

EXPLANATION
Silicosis has been a well-known and proven predisposition to the development of tuberculosis. Due to the increased load on the heart, the formation of cor pulmonale is also a typical complication of advanced or long term silicosis.

195
Q

PBH - 255
Cancer disease (s) caused by asbestos exposure
1) lymphoma
2) Mesothelioma
3) pancreatic cancer
4) lung carcinoma

A) Answers 1, 2 and 3 are correct
B) Answers 1 and 3 are correct
C) Answers 2 and 4 are right
D) only answer 4 is correct
E) all 4 answers are correct

A

ANSWER
C) Answers 2 and 4 are right

EXPLANATION
The relationship between asbestos exposure and mesothelioma is important because mesothelioma is considered as a rare tumor and almost exclusively develops due to exposure to asbestos. Asbestos can also cause lung cancer, but it is not associated with the development of lymphoma and pancreatic cancer.

196
Q

PBH - 256
Benzene:
1) damages the bone marrow and causes aplastic anaemia
2) its vapors mainly damage the lungs
3) due to its chronic effect acute myeloid leukemia may develop
4) causes skin burns

A) answers 1, 2 and 3 are correct
B) answers 1 and 3 are correct
C) answers 2 and 4 are correct
D) only answer 4 is correct
E) all of the answers are correct

A

ANSWER
B) answers 1 and 3 are correct

EXPLANATION
Chronic benzene exposure damages the hematopoietic system, causes aplastic anemia and reversible pancytopenia. Although the production of different cell types is damaged at different level, all hematopoietic changes may indicate benzene toxication. The most serious complications of chronic benzene exposure are the acute or possibly chronic myeloid leukemia.

197
Q

PBH - 257
Which pesticides inhibit/block the activity of acetylcholinesterase?
1) methylcarbamates
2) chlorinated hydrocarbons
3) organic phosphate esters
4) phenol derivatives

A) Answers 1, 2 and 3 are correct
B) Answers 1 and 3 are correct
C) Answer 2 and 4 are correct
D) only answer 4 is correct
E) all 4 answers are correct

A

ANSWER
B) Answers 1 and 3 are correct

EXPLANATION
The carbamate pesticides inhibit of acetylcholinesterase reversibly, while the organic phosphate esters inhibit the enzyme irreversibly. Therefore, carbamate and thiocarbamate poisoning are generally less severe than poisoning caused by organic phosphate esters, but both are dose-dependent.

198
Q

PBH - 258
What are the characteristics of PCBs?
1) they accumulate in the adipose tissue
2) they are excreted in breast milk
3) they induce microsomal enzymes in the liver
4) the population is mainly exposed to them by inhalation

A) Answers 1, 2 and 3 are correct
B) Answers 1 and 3 are correct
C) Answer 2 and 4 are correct
D) only answer 4 is correct
E) all 4 answers are correct

A

ANSWER
A) Answers 1, 2 and 3 are correct

EXPLANATION
PCBs accumulate in the food chain due to their accumulation in animal and human adipose tissue, and are excreted in breast milk. They induce the microsomal enzymes of the liver. The population is mainly exposed to PCBs orally by means of food.

Polychlorinated biphenyls (PCBs) are persistent environmental pollutants that are found at elevated concentrations in the adipose tissue o

199
Q

PBH - 259
Which metabolite detected in the urine refers to benzene exposure during biomonitoring?
1) phenolic acid
2) hippuric acid
3) muconic acid
4) ortho-cresol

A) Answers 1, 2 and 3 are correct
B) Answers 1 and 3 are correct
C) Answers 2 and 4 are correct
D) only answer 4 is correct
E) all 4 answers are correct

A

ANSWER
B) Answers 1 and 3 are correct

EXPLANATION
Periodic medical examination of those working under benzene exposure involves biomonitoring based on metabolites that includes measuring the urinary concentrations of phenol and muconic acid.

200
Q

PBH - 260
What are the stochastic effects of ionizing radiation?
1) teratogenic effects
2) cataract
3) dermatitis caused by radiation
4) carcinogenic effect

A) answers 1, 2 and 3 are correct
B) answers 1 and 3 are correct
C) answers 2 and 4 are correct
D) only answer 4 is correct
E) all of the answers are correct

A

ANSWER
D) only answer 4 is correct

EXPLANATION
Stochastic effects are those effects the frequency and probability of which increses on elevating the doses, more precisely the dose equivalent value. There is no threshold dose in the case of these effects, and the dose-response curve runs through the origo (zero point). A typical example of the stochastic effects is the carcinogenic effect.

201
Q

PBH - 261
What are the characteristics of acute acoustic trauma?

1) The role of air pressure change is significant in its development
2) Developed due sound of high intensity
3) Hearing loss is bilateral
4) Hearing impairment is perceived

A) Answers 1, 2 and 3 are correct
B) Answers 1 and 3 are correct
C) Answers 2 and 4 are right
D) only answer 4 is correct
E) all 4 answers are correct

A

ANSWER
C) Answers 2 and 4 are right

EXPLANATION
Acute acoustic trauma develops due to a single high intensity noise which is not accompanied by significant pressure fluctuation. Therefore, only the inner ear is being damaged and the outer and middle ear will show no abnormal changes. The acoustic damage is perceptual, irreversible, effects the high-frequency sound range as one-sided due to the shielding effect of the head. Mostly it is caused by the firing of firearms.

202
Q

PBH - 264
What is the relationship between the level of education and the health status?
1) The health status of higher educated persons is better than the health status of lower educated persons.
2) Higher educated people are less likely to visit their doctor in severe complaints than those with lower education level.
3) Higher education means a higher comprehensive capacity, therefore it facilitates the access to disease prevention information.
4) The proportion of smokers is higher among higher educated people.

A) answers 1, 2 and 3 are correct
B) answers 1 and 3 are correct
C) answers 2 and 4 are correct
D) only answer 4 is correct
E) all of the answers are correct

A

ANSWER
B) answers 1 and 3 are correct

EXPLANATION
The level of education, like the social status, is an important influencing factor of health status, as it has a positive effect on most indicators describing health status. The positive effects of education on health status are achieved through various mechanisms: higher educated people acquired more (background) knowledge during their studies; by the help of their comprehensive skills they can find and understand better health-related knowledge; because of their wider access to resources, they can reduce their health risks better and, if necessary, have access to health services relatively easier.

203
Q

PBH - 266
What are the risk factors of preterm birth?
1) young age of the mother (under 19 years)
2) lower education level of the mother
3) previous abortion
4) diabetic mother

A) answers 1, 2 and 3 are correct
B) answers 1 and 3 are correct
C) answers 2 and 4 are correct
D) only answer 4 is correct
E) all of the answers are correct

A

ANSWER
E) all of the answers are correct

EXPLANATION
Preterm birth is more than five times higher among women with elementary school qualification than in mothers with a university degree. It is more common among adolescent mothers. Risk factors of preterm birth are the followings: previous artificial abortion, multiple earlier pregnancies, infections, diabetes and hypertension as chronic non-communicable diseases.

204
Q

PBH - 271
What are the most frequent sensory changes affecting the elderly?
1) Cataract
2) Presbyacusis
3) Presbyopia
4) Deterioration of the equilibrium

A) answers 1, 2 and 3 are correct
B) answers 1 and 3 are correct
C) answers 2 and 4 are correct
D) only answer 4 is correct
E) all 4 answers are correct

A

ANSWER
E) all 4 answers are correct

EXPLANATION
The incidence of those diseases is high in the elderly, which are not so important as the direct cause of death but severely impair the life quality. The most common sensory changes are the ones effecting the vision (refraction disorders - presbyopia, cataract), hardness of hearing (presbyacusis), and loss of equilibrium (vestibular disorders).

205
Q

PBH - 274
Which of the following substances is/are responsible for the carcinogenic effect of smoking?
1) N’-nitrosonornicotine
2) nicotine
3) benzo(a)pyrene
4) carbon monoxide

A) Answers 1, 2 and 3 are correct
B) Answers 1 and 3 are correct
C) Answers 2 and 4 are correct
D) Only answer 4 is correct
E) All of the answers are correct

A

ANSWER
B) Answers 1 and 3 are correct

EXPLANATION
Chemical components of tar are responsible for the carcinogenic effects of smoking, including nitrosamines (eg nitrosonornicotine) and benzpyrene, which have been declared carcinogens based on data obtained from International Agency for Research on Cancer (IARC).

206
Q

PBH - 277
Symptoms of vitamin A deficiency:
1) blindness
2) xerophtalmia
3) keratomalacia, complete blindness
4) increased susceptibility to infectious diseases

A) answers 1, 2 and 3 are correct
B) answers 1 and 3 are correct
C) answers 2 and 4 are correct
D) only answer 4 is correct
E) all 4 answers are correct

A

ANSWER
E) all 4 answers are correct

EXPLANATION
Vitamin A is essential for the following functions: vision, intactness of cornea, skin and mucous membranes, reproduction and growth. In the absence of vitamin A hemeralopia, xerophthalmia, keratomalacia, complete blindness as well as skin and mucosal lesions could occur. The immune system is damaged, infections are more common and the outcomes are more severe, growth slows down.

207
Q

PBH - 280
Which of the followings could cause vitamin B12 deficiency?:
1) a strict vegetarian diet
2) Folic acid deficiency
3) atrophic gastritis affecting the elderly
4) lack of UV exposure

A) 1, 2 and 3 is correct
B) the correct answers are 1 and 3
C) 2 and 4 is correct
D) only 4 correct answer
E) 4 and the answer is correct

A

ANSWER
B) the correct answers are 1 and 3

EXPLANATION
In the absence of vitamin B12 (cobalamin), megaloblastic anemia and severe neuropathy develop. Vitamin B12 can only be synthesized by microorganisms. In humans, cobalamin is absorbed in the lower part of the ileum, with the help of gastric intrinsic factor secreted by the stomach. Atrophic gastritis is common among the elders, which causes vitamin B12 deficiency due to decreased secretion of gastric juice. All food of animal origin (liver, meat, fish, egg, milk and dairy products) contain vitamin B12, but plant foods do not contain vitamin B12, therefore a vegetarian diet, especially the rather rigorous vegan diet posesa risk of vitamin B12 deficiency. Folic acid deficiency does not cause the symptoms of vitamin B12 deficiency, but in the absence of vitamin B12, no formation of biologically active tetrahydrofolic acid occurs.

208
Q

PBH - 281
Factors that increase the risk of obesity:
1) sedentary lifestyle
2) poor socio-economic status (in developed countries)
3) high consumption of soft drinks
4) foods with low glycemic index

A) answers 1, 2 and 3 are correct
B) answers 1 and 3 are correct
C) answers 2 and 4 are correct
D) only answer 4 is correct
E) all of the answers are correct

A

ANSWER
A) answers 1, 2 and 3 are correct

EXPLANATION
The main factors increasing the risk of obesity are the sedentary lifestyle, the increased consumption of high-energy foods, the increased consumption of fast-food menus and sweetened soft drinks. Regular exercises and a home/school environment supporting healthy alimentation help to maintain optimal body weight. Regular consumption of foods with low glycemic index (legumes, vegetables, whole grains) are considered as protective factor in the aspect of obesity. Especially high prevalence of obesity could be seen among women with poor social status in developed countries.

209
Q

PBH - 282
Obesity increases the risk of:
1) hypertension
2) Type 2 diabetes
3) breast cancer
4) colorectal cancer

A) answers 1, 2 and 3 are correct
B) answers 1 and 3 are correct
C) answers 2 and 4 are correct
D) only answer 4 is correct
E) all of the answers are correct

A

ANSWER
E) all of the answers are correct

EXPLANATION
According to the World Health Organization, overweight and obesity are the second most important preventable risk factors after smoking. They play an important causal role in the development of hypertension, cardiovascular diseases, cerebral vascular diseases (stroke), type 2 diabetes. It’s a risk factor for many cancers (breast, endometrium, kidney, colorectal carcinoma, pancreas, esophagus).

210
Q

PBH - 283
To prevent the development of type 2 diabetes, the followings is/are recommended:
1) Weight loss
2) Regular physical exercise
3) Consumption of dietary fibers
4) Limiting the intake of saturated fats

A) Answers 1, 2 and 3 are correct
B) Answer 1 and 3 are correct
C) Answers 2 and 4 are correct
D) only 4 is correct answer
E) all 4 answers are correct

A

ANSWER
E) all 4 answers are correct

EXPLANATION
Type 2 diabetes develops through the interaction of genetic and environmental factors. A typical diabetinogenic diet has high energy density, contains high amount of saturated fat and low amounts of dietary fibers. In epidemiological studies strong correlation was found between the increase of body mass index (especially central obesity) and the development of type 2 diabetes. Worldwide, 44% of diabetes risk is attributed to overweight and obesity. Daily intense physical activity or sports activity increases insulin sensitivity and thus reduces the risk of diabetes.

211
Q

PBH - 285
Nutritional recommendation for cancer prevention:
1) Cutdown of alcohol consumption
2) Breastfeeding
3) increased consumption of fresh fruits and vegetables
4) Avoiding milk consumption

A) answers 1, 2 and 3 are correct
B) Answer 1 and 3 are correct
C) 2 and 4 are correct
D) only 4 is correct answer
E) all 4 answers are correct

A

ANSWER
A) answers 1, 2 and 3 are correct

EXPLANATION
Excessive alcohol consumption increases the risk of oral, pharyngeal, laryngeal, esophagus, liver, colon, rectum and breast cancer, therefore cutdown of alcohol consumption is recommended. Breastfeeding has a positive effect on the mother as it reduces breast cancer risk and is beneficial to the baby as it reduces the risk of adult obesity and therefore the development of many cancers. The beneficial effect of increased fruit and vegetable consumption is due to the fact that they contain high quantities of dietary fibers, vitamins, minerals and other phytochemicals. The protective effect of milk consumption was observed against the development of colorectal cancer.

212
Q

PBH - 288
In case of a suspected mushroom poisoning:
1) the patient should be vomited or gastric lavage must be performed
2) persons suspected of mushroom poisoning should be observed for 48 hours
3) stomach lavage liquid, food leftovers, kitchen wastes should be sent for laboratory testing
4) the Public Health Authority should be notified in a written reporting form

A) answers 1, 2 and 3 are correct
B) answers 1 and 3 are correct
C) answers 2 and 4 are correct
D) only answer 4 is correct
E) all 4 answers are correct

A

ANSWER
E) all 4 answers are correct

EXPLANATION
In case of mushroom poisoning the main components of the first aid are the induction of vomiting and / or performing gastric lavage as soon as possible after the consumption of mushrooms. People suspected of having mushroom poisoning are also advised to be hospitalized even in case of having only mild symptoms because the person who picked the mushrooms could collect other species too, including not only moderately harmful ones with short latency and early symptoms but also lethal ones with longer latency (eg death-cup). If poisoning is suspected, further consumption of the food containing mushroom components must be prohibited, the rest of the food, the kitchen cleaning waste, a portion of the gastric lavage fluid and the vomit, respectively should be sent for laboratory testing. For the prevention of mushroom-induced poisoning the most important task is the examination of the collected mushrooms by a mushroom expert.

213
Q

PBH - 291
Which of the following activities is/are the most popular method(s) of quality improvement (PDCA)?
1) Planning
2) Action
3) Check (control)
4) Feedback

A) answers 1, 2 and 3 are correct
B) answers 1 and 3 are correct
C) answers 2 and 4 are correct
D) only answer 4 is correct
E) all 4 answers are correct

A

ANSWER
E) all 4 answers are correct

EXPLANATION
The PDCA acronym used in quality improvement consists of the initials of the English words plan, do, check, act. It refers to the quality improvement cycle that consists of planning (necessary changes, preparation and planning of interventions), action (implementation of changes), control (impact assessment) and feedback.

214
Q

PBH - 293
Which of the followings is/are the main fields of health policies?
1) Supporting health oriented decisions to be made
2) Ensuring equal access to care
3) Health protection and health promotion
4) Preventing special health problems

A) Answers 1, 2 and 3 are correct
B) Answers 1 and 3 are correct
C) Answers 2 and 4 are correct
D) only 4 is correct answer
E) all 4 answers are correct

A

ANSWER
E) all 4 answers are correct

EXPLANATION
All 4 areas listed in the question (support for health-oriented decision-making in all public areas, ensuring equal access to health care for all, health protection ad promotion, prevention, detection and treatment of specific health problems) are the main areas of health policies in most countries of the world. It should be noted that the term policy in health policy is used as the way to achieve certain goals.

215
Q

PBH - 294
Which of the following statements applies to the solidarity-type insurance system?
1) Ensures equal opportunities better compared to the risk system
2) It is profit-oriented system
3) It does not encourage participants to maintain disease-preventive lifestyles
4) This system provides health services of the highest quality

A) answers 1, 2 and 3 are correct
B) answers 1 and 3 are correct
C) answers 2 and 4 are correct
D) only answer 4 is correct
E) all 4 answers are correct

A

ANSWER
B) answers 1 and 3 are correct

EXPLANATION
The essence of an insurance system based on solidarity (also called Bismarck’s insurance system), is that everyone pays contribution in proportion to their income, and when they are sick, they receive care that meet their needs. The system is therefore not for-profit and basically offers equal possibilities to the contributors, as the benefit does not depend on the amount of the contribution made. At the same time, contributors have no interest in health preserving lifestyle because they have no financial benefits in regard to neither contributions paid nor services used. The system is unable to provide the most up-to-date (and most expensive) care options for all, as this can not be financed by the contributions.

216
Q

PBH - 295
Act LXXXIII (1997) (about the mandatory health insurance in Hungary) applies which principles to the regulation of health insurance from the followings?
1) Financial health insurance services are proportional to the amount of the mandatory health insurance contribution
2) Healthcare services have the same professional content for all people who are entitled to health care
3) The State ensures that the services specified in this Act be fulfilled even if the necessary costs can not be covered by the health insurance
4) In the structure of the health care system, progressivity has the utmost importance

A) answers 1, 2 and 3 are correct
B) Answer 1 and 3 are correct
C) 2 and 4 are correct
D) only 4 is correct answer
E) all 4 answers are correct

A

ANSWER
A) answers 1, 2 and 3 are correct

EXPLANATION
Act LXXXIII of 1997 lays down five principles for the regulation of health insurance in Hungary: health services can be used to an extent justified by the health conditions under the law (the service is independent of the amount of the contribution); Cash benefits may be used in proportion to the health insurance contribution (eg sickness benefit); Healthcare services offer the same professional services to all people entitled to healthcare (equal opportunities); the state ensures the fulfillment of the statutory services from the state budget, even if the necessary expenses cannot be covered by the health insurance (state commitment); the authorities carrying out the health insurance tasks inform the insured persons of their rights and obligations and assist in enforcing their claims (right to information). The principle of progressivity does not appear here, it can be found among the principles of the act CLIV (1997) regarding health care.

217
Q

REMOVED FROM 2024!!!

PBH - 296
For which of the following areas and institutions, respectively lies the responsibilities on State Secretariat for Healthcare of the Ministry of Human Capacities
1) National Public Health and Medical Office Service
2) Patient care at universities
3) The authorities of the health care with national power
4) WHO Regional Office

A) answers 1, 2 and 3 are correct
B) Answer 1 and 3 are correct
C) Answer 2 and 4 are correct
D) only answer 4 is correct
E) all 4 answers are correct

A

ANSWER
A) answers 1, 2 and 3 are correct

EXPLANATION
While the first three areas / institutions are the responsibility of the State Secretariat for Healthcare, the Regional Office of WHO is not supervised by them, but they coordinate the cooperation with it.

218
Q

PBH - 297
Which of the followings are among the main objectives of the European Health Strategy 2008-2013?
1) Harmonization of the health policies of the member states
2) Protecting the health of the aging population
3) Establishment of a Joint European Health Fund to finance the EU action programs
4) Support of dynamic health systems and new technologies for the sustainable development of health systems

A) 1, 2 and 3 are correct
B) Answer 1 and 3 are correct
C) 2 and 4 are correct
D) only 4 correct answer
E) all 4 answers are correct

A

ANSWER
C) 2 and 4 are correct

EXPLANATION
The European Union has implemented a number of joint action programs in the field of health in the context of closer integration. The EU published a White Paper in 2007 estabilishing the health strategy for 2008-2013. The three main objectives of this publication were: protecting health in the aging Europe, protecting citizens from health threats, supporting dynamic health care systems and supporting new technologies.

219
Q

PBH - 321
Interventions aimed at cardiovascular disease and cancer prevention are often the same, because the etiological backgrounds of the two diseases significantly overlap.
A) both are right, they have a causal relationship between them
B) both are correct, but there is no cause-and-effect relationship between them
C) the first is correct in itself, the second is wrong
D) The first is wrong, the second is correct in itself
E) both are wrong

A

ANSWER
A) both are right, they have a causal relationship between them

EXPLANATION
Both statements are correct, and there is a cause-and-effect relationship between them. A large-scale prevention program very often cause significant improvements in the incidence/prevalence of multiple diseases. North Karelia programme could be consideres as an example, because its main aim was to reduce the rate of premature death due to cardivasvular diseases and was launched by the Finnish government in the early 1970s. Since the program - among other things - was aimed at promoting a healthy diet and the reduction of smoking, it has made significant improvements in deaths caused by cancer-related diseases (more than 25 years after the start of the program, premature death resulting from cardiovascular diseases decreased by 60% and premature deaths due to cancer decreased by 40%).

220
Q

PBH - 322
Breast cancer mortality rates among Hungarian women are more than twice as high as the EU-15 average,because in Hungary breast cancer screeningwas introduced only in 2005.
A) both are right, they have a causal relationship between them
B) both are correct, but there is no cause-and-effect relationship between them
C) the first is correct in itself, the second is wrong
D) The first is wrong, the second is correct in itself
E) both are wrong

A

ANSWER
E) both are wrong

EXPLANATION
The first and second statements are both wrong. Breast cancer mortality among Hungarian women exceeded the EU15 average by 14% in 2007. Breast cancer screening was introduced in Hungary in 2001.

221
Q

PBH - 323
Breast cancer and ovarian cancer sometimes have familial accumulations because in these cancers the background of genetic susceptibility is partly the same.
A) both are right, they have a causal relationship between them
B) both are correct, but there is no cause-and-effect relationship between them
C) the first is correct in itself, the second is wrong
D) The first is wrong, the second is correct in itself
E) both are wrong

A

ANSWER
A) both are right, they have a causal relationship between them

EXPLANATION
Both parts of the statement are correct, and there is a cause-and-effect relationship between them. Breast cancer and ovarian cancer cases show occasionally familial accumulation that is caused by mutations of the BRCA1 / BRCA2 genes.

222
Q

PBH - 324
The potential carcinogenic effect of androgens has came up among bodybuilders, because an increased rate of incidence of prostate cancer could be seen among them.
A) both are right, they have a causal relationship between them
B) both are correct, but there is no cause-and-effect relationship between them
C) the first is correct in itself, the second is wrong
D) The first is wrong, the second is correct in itself
E) both are wrong

A

ANSWER
C) the first is correct in itself, the second is wrong

EXPLANATION
The potential carcinogenic effect of androgens came up due to an increased incidence of hepatoma (and not prostate cancer) in bodybuilders, but the correlation was not confirmed either in this case.

223
Q

PBH - 325
Women in the premenopausal period have a lower risk of hypertension than men of the same age, becausein women the effects of the predisposing genes become effective later.
A) both are right, they have a causal relationship between them
B) both are correct, but there is no cause-and-effect relationship between them
C) the first is correct in itself, the second is wrong
D) The first is wrong, the second is correct in itself
E) both are wrong

A

ANSWER
B) both are correct, but there is no cause-and-effect relationship between them

EXPLANATION
Both parts of the statement are correct, but there is no cause-and-effect relationship between them. The fact that women in the premenopausal period are less prone to hypertension can be explained by the protective effect of estrogens. The phenomenon that the effect of predisposing genes in women becomes effective later is also due to the estrogens.

224
Q

PBH - 327
The incidence of obesitity has increased dramatically in most developed countries, because a diet which replaces saturated fats with monounsaturated fats has no cholesterol-lowering effect.
A) both are right, they have a causal relationship between them
B) both are correct, but there is no cause-and-effect relationship between them
C) the first is correct in itself, the second is wrong
D) The first is wrong, the second is correct in itself
E) both are wrong

A

ANSWER
B) both are correct, but there is no cause-and-effect relationship between them

EXPLANATION
Both parts of the statement are correct, but there is no cause-and-effect relationship between them. The increase in the prevalence of obesity in the industrialized countries can be explained by the fact that a positive change in the social status first causes changes in dietary habits and the prevalence of obesity increases due to the spread of high-calory diets.

225
Q

PBH - 330
Chronic liver diseases and cirrhosis are the most common cause of deaths among men in Hungary, because more than 80% of deaths dueto gastrointestinal diseases are caused by liver diseases.
A) both are right, they have a causal relationship between them
B) both are correct, but there is no cause-and-effect relationship between them
C) the first is correct in itself, the second is wrong
D) The first is wrong, the second is correct in itself
E) both are wrong

A

ANSWER
D) The first is wrong, the second is correct in itself

EXPLANATION
The most common causes of deaths are the cardiovascular diseases in Hungary, but it is true that more than 80% of deaths due to gastrointestinal diseases are caused by liver diseases in Hungary.

226
Q

PBH - 331
The reason behind about 80 % of gastric ulcers is an infection with H. pylori, because according to the estimations 80% of the world’s population are infected with this pathogen.
A) both are right, they have a causal relationship between them
B) both are correct, but there is no cause-and-effect relationship between them
C) the first is correct in itself, the second is wrong
D) The first is wrong, the second is correct in itself
E) both are wrong

A

ANSWER
C) the first is correct in itself, the second is wrong

EXPLANATION
Gastric ulcers develop in about 80% of the cases due to H. pylori infection, but it is estimated that more than half of the world’s population (about 65% of the population of Hungary) is infected with this pathogen.

227
Q

PBH - 333
25% of the population develop a mental illness or a behavioral disorder during their lifetime, because the most frequent disease in this group is depression.
A) both are right, they have a causal relationship between them
B) both are correct, but there is no cause-and-effect relationship between them
C) the first is correct in itself, the second is wrong
D) The first is wrong, the second is correct in itself
E) both are wrong

A

ANSWER
B) both are correct, but there is no cause-and-effect relationship between them

EXPLANATION
Both parts of the statement are correct, but there is no cause-effect relationship between them. The World Health Organization (WHO) estimates that 25% of the population in developed and developing countries exhibit mental illnesses or behavioral disorders during their lifetime. The most frequent disorders within this disease group are depression, schizophrenia and Alzheimer’s disease.

228
Q

PBH - 335
Vaccination is one of the most important primary prevention tool, because the use of vaccines significantly influences the incidence of infectious diseases.
A) both are right, they have a causal relationship between them
B) both are correct, but there is no cause-and-effect relationship between them
C) the first is correct in itself, the second is wrong
D) The first is wrong, the second is correct in itself
E) both are wrong

A

ANSWER
B) both are correct, but there is no cause-and-effect relationship between them

EXPLANATION
The statement is true because vaccines are effective in the prevention of infectious diseases. The reasoning is also correct, but does not explain the statement, rather the relationship is vice versa: Vaccines belong to primary prevention tools (based on the definition of primary prevention) because they reduce the incidence of infectious diseases.

229
Q

PBH - 336
Pregnancy is an absolute contraindication to the administration of vaccines because the vaccine may also affect the fetus.
A) both are right, they have a causal relationship between them
B) both are correct, but there is no cause-and-effect relationship between them
C) the first is correct in itself, the second is wrong
D) The first is wrong, the second is correct in itself
E) both are wrong

A

D) The first is wrong, the second is correct in itself

EXPLANATION
Because the vaccine may have effects on the fetus, pregnant women will only be vaccinated if it is certain that the vaccine will not cause any complications to the fetus, or if there is a serious situation (eg. rabies) so that it is absolutely necessary to vaccinate the mother. During pregnancy, live vaccines are the most dangerous, toxoids or genetically modified vaccines are generally harmless. Do not use inactivated vaccines as they are much more reactive than the toxoid type (although there is a difference between the reactogenicity of each vaccine).

230
Q

PBH - 337
If 10 years or more have elapsed since the last revaccination and the wound is severely damaged, contaminated with soil, then the patient should be given active and passive immunization against tetanus because there is an increased risk of infection in these cases.
A) both are right, they have a causal relationship between them
B) both are correct, but there is no cause-and-effect relationship between them
C) the first is correct in itself, the second is wrong
D) The first is wrong, the second is correct in itself
E) both are wrong

A

ANSWER
A) both are right, they have a causal relationship between them

EXPLANATION
The prevention of tetanus is achieved by age-related mandatory vaccination. If atetanus infection is suspected (broken, torn, deeply soiled wound, etc.), the victim should be vaccinated in addition to the appropriate surgical care as follows. If they received both primary immunization and revaccination and the last vaccination was received within 5 years, a vaccination is not required; when 5 or more years have passed, tetanus toxoid should be given. ; If 10 years or more have passed since the last revaccination and the wound is heavily torn, contaminated with soil, contains a foreign body or the patient has lost blood, is in shock, a large area ofthe skin is burned, is damaged with nuclear radiation, than both active and passive immunization (toxoid and human tetanus immunoglobulin) is required. B - both are correct, but there is no cause-and-effect relationship between them

231
Q

PBH - 338
In case of suspected acute hepatitis B virus infection, passive immunization makes no sense at all because due to the low occurence of the population, the γ-globulin products are low on titer.
A) both are right, they have a causal relationship between them
B) both are correct, but there is no cause-and-effect relationship between them
C) the first is correct in itself, the second is wrong
D) The first is wrong, the second is correct in itself
E) both are wrong

A

ANSWER
D) The first is wrong, the second is correct in itself

EXPLANATION
In case of suspected hepatitis infection, the person should preferably be passively immunized within 24 hours. A specific immunoglobulin-containing vaccine (eg HBIG) should be used, γ-globulin formulas are insufficient because these do not contain enough antibodies to hepatitis B due to the low rate of infection of the population. In general, it should be noted that γ-globulin reflects the overall antibody level of the general population, so it can be effective against diseases (eg, hepatitis A) that infected a significant proportion of the population (this is also possible in asymptomatic form).

232
Q

PBH - 339
The incubation period of Clostridium perfringens toxin infection is 6 to 24 hours, because the disease is characterized by abdominal cramping, multiple, often unbound diarrhoea and rapid recovering.

A) both are right, they have a causal relationship between them
B) both are correct, but there is no cause-and-effect relationship between them
C) the first is correct in itself, the second is wrong
D) The first is wrong, the second is correct in itself
E) both are wrong

A

ANSWER
B) both are correct, but there is no cause-and-effect relationship between them

EXPLANATION
Both statements are true, but there is no causal relationship between them. It is noteworthy that Clostridium perfringens produces its toxins in the gastrointestinal tract and not in food.

233
Q

PBH - 340
Nowadays iodine-containing disinfectants are not used because they sometimes cause allergic symptoms.
A) both are right, they have a causal relationship between them
B) both are correct, but there is no cause-and-effect relationship between them
C) the first is correct in itself, the second is wrong
D) The first is wrong, the second is correct in itself
E) both are wrong

A

ANSWER
D) The first is wrong, the second is correct in itself

EXPLANATION
Iodine-based disinfectants are still widely used today, but unlike in earlier years, they contain varoius complex iodine compounds instead of iodine tincture. They are non-irritant but may cause allergic reactions in certain cases.

234
Q

PBH - 341
The reporting of infectious diseases (regardless of the submission of theregistration form) should always also be done over the phone, because this is the only effective way to prevent the spread of infectious diseases.
A) both are right, they have a causal relationship between them
B) both are correct, but there is no cause-and-effect relationship between them
C) the first is correct in itself, the second is wrong
D) The first is wrong, the second is correct in itself
E) both are wrong

A

ANSWER
E) both are wrong

EXPLANATION
Only the most serious, highly infectious diseases must be reported by telephone calls (diseases listed in the regulation). Most infectious diseases that have to be reported do not require immediate actions by the epidemiological authorities which would indicate telephone notification.

235
Q

PBH - 342
Boiling (cooking) foods prevents food poisoning caused by Staphylococcus aureus because the S. aureus’ toxin is sensitive to heat.
A) both are right, they have a causal relationship between them
B) both are correct, but there is no cause-and-effect relationship between them
C) the first is correct in itself, the second is wrong
D) The first is wrong, the second is correct in itself
E) both are wrong

A

ANSWER
E) both are wrong

EXPLANATION
The toxin of Staphylococcus aureus is heat resistant so the disease can not be prevented by boiling foods.

236
Q

PBH - 344
In patients infected with syphilis and gonorrhea, an HIV screening test should also be performed because a single person may has been infected with several sexually transmitted diseases at the same time.
A) both are right, they have a causal relationship between them
B) both are correct, but there is no cause-and-effect relationship between them
C) the first is correct in itself, the second is wrong
D) The first is wrong, the second is correct in itself
E) both are wrong

A

ANSWER
A) both are right, they have a causal relationship between them

EXPLANATION
In the case of sexually transmitted diseases, it is possible that in homosexual / heterosexual relationships, one of the partners has already been infected with several pathogens, thus these can be transmitted simultaneously to the unaffected (healthy) partner. It should be noted that certain pathogens (HBV, HCV, HIV) that belong to the STD group can not only be transmitted sexually (eg common use of needles among drug users). This possibility, however, does not modify the mandatory HIV screening of patients with syphilis and gonorrhea.

237
Q

PBH - 346
The proportion of reported nosocomial infections as a result of preventive interventions/processes decreased to 4-5% in Hungary, because nosocomial surveillance with adequate method and intensity can prevent approximately one third of these infections.
A) both are right, they have a causal relationship between them
B) both are correct, but there is no cause-and-effect relationship between them
C) the first is correct in itself, the second is wrong
D) The first is wrong, the second is correct in itself
E) both are wrong

A

ANSWER
D) The first is wrong, the second is correct in itself

EXPLANATION
The first part of the sentence is incorrect because, in contrast to the internationally accepted 4-5% level of nosocomial infections, only a 1.5% value of these infections are reported in Hungary. The statement which is described in the second sentence is correct, because monitoring according to professional requirements can actually prevent 30% of such infections.

238
Q

PBH - 347
Not all HIV-positive pregnant women give birth to an infected newborn because the transplacental transmission of the virus from the mother to the fetus occurs in 30-40% of the cases.
A) both are right, they have a causal relationship between them
B) both are correct, but there is no cause-and-effect relationship between them
C) the first is correct in itself, the second is wrong
D) The first is wrong, the second is correct in itself
E) both are wrong

A

ANSWER
A) both are right, they have a causal relationship between them

EXPLANATION
Both parts of the sentence are correct, as only 30-40% of HIV-positive pregnant women infect the fetus via the placenta. It should be noted, that the fetus may be infected during delivery and may also be infected with breastmilk containing the virus of the HIV-positive mother. In the case of an HIV-negative fetus both infection forms may be and has to be prevented.

239
Q

PBH - 348
Varicella infection in the first trimester of pregnancy causes symptoms only in the mother, because the varicella virus is not transferred transplacentally to the fetus.
A) both are right, they have a causal relationship between them
B) both are correct, but there is no cause-and-effect relationship between them
C) the first is correct in itself, the second is wrong
D) The first is wrong, the second is correct in itself
E) both are wrong

A

ANSWER
E) both are wrong

EXPLANATION
Both statements are wrong. The varicella virus can actually cause symptoms among pregnant women (eg skin rash, pneumonia), but can also be transferred to the fetus and may cause a congenital varicella syndrome with severe complex developmental disorders.

240
Q

PBH - 350
Chemical safety aims at protecting human health and the environment against the hazards of chemicals, because as the first phase of the process, the substances should be classified on the basis of their physical, physicochemical, human- and ecotoxicological properties .
A) both are right, they have a causal relationship between them
B) both are correct, but there is no cause-and-effect relationship between them
C) the first is correct in itself, the second is wrong
D) The first is wrong, the second is correct in itself
E) both are wrong

A

ANSWER
B) both are correct, but there is no cause-and-effect relationship between them

EXPLANATION
Both parts of the sentence are correct, but there is no correlation between them. The primary goal of chemical safety is the protection of human health and the environment from the dangers caused by chemicals, and the first phase is categorizing them based on their actual physical, physico-chemical, human- and ecotoxicological properties.

241
Q

PBH - 351
As an effect of the global warming summer heatwaves cause more cardiovascular and respiratory illness-related deaths because primarily elderly people suffering from these chronic diseases tolerate poorly the climatic effects of heatwaves and among them the lethal outcome is more frequent.
A) both are right, they have a causal relationship between them
B) both are correct, but there is no cause-and-effect relationship between them
C) the first is correct in itself, the second is wrong
D) The first is wrong, the second is correct in itself
E) both are wrong

A

ANSWER
A) both are right, they have a causal relationship between them

EXPLANATION
Both parts of the statement are correct and there is a correlation between them. Global warming could cause heatwaves, and these climatic features, especially the extreme high temperatures are not well tolerated by elderly patients suffering from chronic cardiovascular/respiratory diseases. As a result, the number of deaths is increases, especially among them.

242
Q

PBH - 352
The symptoms of acute and the chronic exposure of xenobiotics are the same because their point of action and mechanism of action are the same in both forms of intoxication.
A) both are right, they have a causal relationship between them
B) both are correct, but there is no cause-and-effect relationship between them
C) the first is correct in itself, the second is wrong
D) The first is wrong, the second is correct in itself
E) both are wrong

A

ANSWER
E) both are wrong

EXPLANATION
Both parts of the statement are wrong. For a significant proportion of xenobiotics, the symptoms of acute and chronic intoxications differ (see, for example, benzene) partly because due to a difference in the duration of the exposure (and the level of the exposure) the same xenobiotic could act on different attack points and thus could evoke different mechanisms.

243
Q

PBH - 353
The amount of pollutants emitted into the air is given in mg / m3 because they may reach different distances from their emission point, depending on geographical and weather conditions.
A) both are right, they have a causal relationship between them
B) both are correct, but there is no cause-and-effect relationship between them
C) the first is correct in itself, the second is wrong
D) The first is wrong, the second is correct in itself
E) both are wrong

A

ANSWER
D) The first is wrong, the second is correct in itself

EXPLANATION
The first part of the statement is false because the dimension of the emission is not mg / m3, but g / h or kg / year. The second part is correct because the particles released could travel from their emission point to various distances, sometimes hundreds of kilometers away.

244
Q

PBH - 354
The alkylated mercury derivatives are stable and lipophilic because they ambient elemental or inorganic mercury is converted to methyl or dimethylmercury due to the biotransformation.
A) both are right, they have a causal relationship between them
B) both are correct, but there is no cause-and-effect relationship between them
C) the first is correct in itself, the second is wrong
D) The first is wrong, the second is correct in itself
E) both are wrong

A

ANSWER
B) both are correct, but there is no cause-and-effect relationship between them

EXPLANATION
Both parts of the statement are correct, but there is no connection between them. Alkylated organic mercury compounds are actually more stable and more lipophilic; because due to their latter property they can be enriched in the food chain. The elemental mercury or its inorganic compounds are converted by biotransformation, especially in aquatic environment, into methyl or dimethylmercury.

245
Q

PBH - 356
The main symptom of chronic human pyrethroid intoxication is paresthesia of peripheral nerves, because their toxic metabolite formed in mammals damages the myelin sheet.
A) both are right, they have a causal relationship between them
B) both are correct, but there is no cause-and-effect relationship between them
C) the first is correct in itself, the second is wrong
D) The first is wrong, the second is correct in itself
E) both are wrong

A

ANSWER
E) both are wrong

EXPLANATION
Both parts of the statement are wrong. Neither a human toxic metabolite of pyrethroid nor symptoms associated with chronic pyrethroid exposure are known.

246
Q

PBH - 357
Women exposed to teratogenic chemicals should be advised to report their lack of menstruation immediately because pregnant women should not be exposed to such toxic effects, so they should be taken off/removed from the given work.
A) both are right, they have a causal relationship between them
B) both are correct, but there is no cause-and-effect relationship between them
C) the first is correct in itself, the second is wrong
D) The first is wrong, the second is correct in itself
E) both are wrong

A

ANSWER
A) both are right, they have a causal relationship between them

EXPLANATION
Both parts of the statement are correct and there is a relationshp between them. Women could not be exposed to teratogenic effects, especially in the early stages of pregnancy, so they should be taken off the given work after the pregnancy is confirmed

247
Q

PBH - 358
Elderly people are the most suceptible to “sick building” syndrome because the symptoms develop due to the increase and accumulation of concentrations of indoor chemical air pollutants.
A) both are right, they have a causal relationship between them
B) both are correct, but there is no cause-and-effect relationship between them
C) the first is correct in itself, the second is wrong
D) The first is wrong, the second is correct in itself
E) both are wrong

A

ANSWER
D) The first is wrong, the second is correct in itself

EXPLANATION
The first part of the statement is incorrect, as children less than 5 years old are the most prone to the Sick Building Syndrome. The second part is correct because the increased concentration and accumulation of indoor chemical pollutants play a role in the development of the syndrome.

248
Q

PBH - 359
PCBs accumulate in adipose tissue and are excreted in breast milk, because their main representatives, the dioxins mean occupational exposure mainly in the electrical and paint industries.
A) both are right, they have a causal relationship between them
B) both are correct, but there is no cause-and-effect relationship between them
C) the first is correct in itself, the second is wrong
D) The first is wrong, the second is correct in itself
E) both are wrong

A

ANSWER
C) the first is correct in itself, the second is wrong

EXPLANATION
The first part of the statement is correct, as lipophilic PCBs are indeed cumulative in animal and human adipose tissue and are excreted in breast milk. The second part is wrong, because the dioxins are not PCBs, but PCDDs and PCDFs, and are not related to the electrical and paint industries, they are pesticides and cause chronic human processes as contaminants.

249
Q

PBH - 360
The composition of karstic water is generally favorable for human consumption because it does not contain any environmental pollution due to its geological location.
A) both are right, they have a causal relationship between them
B) both are correct, but there is no cause-and-effect relationship between them
C) the first is correct in itself, the second is wrong
D) The first is wrong, the second is correct in itself
E) both are wrong

A

ANSWER
C) the first is correct in itself, the second is wrong

EXPLANATION
The first half of the sentence is correct because the composition of karstic waters is generally suitable for human consumption. The second part is false, as the pollutants from the environment are not filtered from the water by geological conditions, thus the natural purification of the karstic water does not happen.

250
Q

PBH - 362
Acute acoustic trauma usually affects the inner ear only because high-intensity noise is not associated with significant air pressure changes.
A) both are right, they have a causal relationship between them
B) both are correct, but there is no cause-and-effect relationship between them
C) the first is correct in itself, the second is wrong
D) The first is wrong, the second is correct in itself
E) both are wrong

A

ANSWER
A) both are right, they have a causal relationship between them

EXPLANATION
Acute acute trauma develops from a single high-intensity sound of more than 125 dB when the high-intensity sound is not associated with significant air pressure changes. Therefore, only the inner ear (mainly the hair cells) is damaged, the examination of the outer and middle ear does not indicate pathological changes.

251
Q

PBH - 363
Acoustic blast trauma induced hearing loss is most commonly caused by the firing of firearms, because the damage is caused by the combined effect of a single, high-intensity sound wave and a change in air pressure.
A) both are right, they have a causal relationship between them
B) both are correct, but there is no cause-and-effect relationship between them
C) the first is correct in itself, the second is wrong
D) The first is wrong, the second is correct in itself
E) both are wrong

A

ANSWER
D) The first is wrong, the second is correct in itself

EXPLANATION
Acoustic blast trauma induced hearing loss is caused by a single-wave, high-intensity sound effect and a change in the air pressure at the same time. Mostly it is caused by an explosion, but rarely occurs as a sports injury (water polo players, handballers, boxers).

252
Q

PBH - 366
The maximum concentration (MC) of carcinogenic substances in the air in the working enviroment is also referred to as “tolerable” value, because concentrations of the carcinogenic substances under the MC value cause no tumour.
A) both are right, they have a causal relationship between them
B) both are correct, but there is no cause-and-effect relationship between them
C) the first is correct in itself, the second is wrong
D) The first is wrong, the second is correct in itself
E) both are wrong

A

ANSWER
C) the first is correct in itself, the second is wrong

EXPLANATION
The MC value of carcinogenic substances is also referred to as “tolerable” value. This emphasizes that, to the best of our knowledge, carcinogenic chemicals do not have a safe threshold concentration or a threshold dose.

253
Q

PBH - 367
Occupational diseases are prevented by primary prevention, because screening is the most effective method of primary prevention.
A) both are right, they have a causal relationship between them
B) both are correct, but there is no cause-and-effect relationship between them
C) the first is correct in itself, the second is wrong
D) The first is wrong, the second is correct in itself
E) both are wrong

A

ANSWER
C) the first is correct in itself, the second is wrong

EXPLANATION
The prevention of occupational diseases is carried out by primary prevention that means the possible elimination or at least attenuation of the risk factors. Even the most organized secondary prevention (screening) has no impact on the incidence of the disease, but recognizes the disease at an early stage when amore effective early treatment could be carried out if needed.

254
Q

PBH - 368
Work-related diseases occur only at workers because work-related diseases can be largely correlated with the profession.
A) both are right, they have a causal relationship between them
B) both are correct, but there is no cause-and-effect relationship between them
C) the first is correct in itself, the second is wrong
D) The first is wrong, the second is correct in itself
E) both are wrong

A

ANSWER
E) both are wrong

EXPLANATION
Diseases that largely correlate with occupations are called occupational diseases. On the other hand, he work environment and its characteristics, effects originating from the work process and its circumstances together with other risk factors can play a role in the development, progression and outcome of complex, multi-causal diseases. These multiple pathophysiological disorders are more common among certain group of workers than among the population in general or among other worker group. These diseases are distinguished from the occupational diseases, they are called work- related diseases.

255
Q

PBH - 369
The effective temperature takes into account the air temperature, the relative air humidity and the speed of the airflow, because the effective temperature can be determined with the help of a nomogram.
A) both are right, they have a causal relationship between them
B) both are correct, but there is no cause-and-effect relationship between them
C) the first is correct in itself, the second is wrong
D) The first is wrong, the second is correct in itself
E) both are wrong

A

ANSWER
B) both are correct, but there is no cause-and-effect relationship between them

EXPLANATION
The effective temperature takes into account the air temperature, relative air humidity and the speed of the airflow rate. The effective temperature can be determined with the help of a nomogram.

256
Q

PBH - 370
Acclimatization to heat work is usually completed within 2 weeks because during the acclimatization the heart rate increases.
A) both are right, they have a causal relationship between them
B) both are correct, but there is no cause-and-effect relationship between them
C) the first is correct in itself, the second is wrong
D) The first is wrong, the second is correct in itself
E) both are wrong

A

ANSWER
C) the first is correct in itself, the second is wrong

EXPLANATION
Particular change in the heat balance is called acclimatization. The essence of acclimatization is that the heat loss becomes more effective (primarily the perspiration increases), while both the core temperature (after an initial temporary increase) and the heart rate decreases during the same physical activity. Acclimatization is usually completed within 2 weeks, and the acclimatized status ceases in approximately the same amount of time.

257
Q

PBH - 371
The acute local radiation injury has a threshold dose, because an acute local radiation injury is the deterministic effect of ionizing radiation.
A) both are right, they have a causal relationship between them
B) both are correct, but there is no cause-and-effect relationship between them
C) the first is correct in itself, the second is wrong
D) The first is wrong, the second is correct in itself
E) both are wrong

A

ANSWER
A) both are right, they have a causal relationship between them

EXPLANATION
Those effects are called deterministic which develop above a certain dose (so-called threshold dose) at everybody. The threshold dose is relatively high, it is ususal that when the dose is increased, the effects become more severe and finally fatal. Acute local radiation injury: exposure of isolated skin to ionizing radiation (3-10 Gy) causes erythema, hair loss, dry exfoliation and moist exfoliation.

258
Q

PBH - 376
Young people are entitled to home pediatrician care until the age of 18, because childhood lasts up to their18th year according to the UN definition.
A) both are right, they have a causal relationship between them
B) both are correct, but there is no cause-and-effect relationship between them
C) the first is correct in itself, the second is wrong
D) The first is wrong, the second is correct in itself
E) both are wrong

A

ANSWER
A) both are right, they have a causal relationship between them

EXPLANATION
The United Nations Charter on the Rights of Children sets the ages limit of childhood between 0 and 18 years Accordingly, children are entitled for paediatrician care if available, until the age of 18 years.

259
Q

PBH - 379
The prevalence of age-related diseases associated with the fray of the skeletal system is high because these diseases cause significant loss of functin thus deteriorating life quality.
A) both are right, they have a causal relationship between them
B) both are correct, but there is no cause-and-effect relationship between them
C) the first is correct in itself, the second is wrong
D) The first is wrong, the second is correct in itself
E) both are wrong

A

ANSWER
B) both are correct, but there is no cause-and-effect relationship between them

EXPLANATION
The prevalence of skeletal system fray disorders (osteoarthritis, osteoporosis, spinal hernia) is high in people older than 65 years and significantly affects mobility, leading to various consequences. According to Hungarian data obtained from the 2009 European Health Interview Survey, two-thirds of people older than 65-years live their lifes with disabilities and 58% of them are unable hardly able to travel 500 meters away on a plain ground without any aids/support.

260
Q

PBH - 380
Regular physical exercise helps maintaining muscle strength among the elderly because it reduces the risk of constipation and mental deterioration.
A) both are right, they have a causal relationship between them
B) both are correct, but there is no cause-and-effect relationship between them
C) the first is correct in itself, the second is wrong
D) The first is wrong, the second is correct in itself
E) both are wrong

A

ANSWER
B) both are correct, but there is no cause-and-effect relationship between them

EXPLANATION
Regular exercise reduces the risk of loosing muscle strength and by exercising the abdominal muscles exerts external pressure on the abdominal organs which aids bowel movement. Accoring to a current summary of the Cochrane Reviews, aerobics-type exercise improves cognitive performance in people over the age of 55. However, the mechanism behind this effect is not known yet.

261
Q

PBH - 381
It is recommended for everyone to get a vaccination against influenza and pneumonia caused by pneumococci over the age of 65, because the mortality of these diseases is high at this age.
A) both are right, they have a causal relationship between them
B) both are correct, but there is no cause-and-effect relationship between them
C) the first is correct in itself, the second is wrong
D) The first is wrong, the second is correct in itself
E) both are wrong

A

ANSWER
A) both are right, they have a causal relationship between them

EXPLANATION
A proven and effective way to prevent influenza and pneumonia is the vaccination. There is an increased risk of developing complications for people over the age of 65 years in both cases, which also increases the risk of death. One vaccinaton is enough against pneumococcal pneumonia, while influenza vaccination is recommended to be repeated yearly.

262
Q

PBH - 384
Deterioration of social status reduces the risk of death in men, because the worsening of social status causes severe chronic stress.
A) both are right, they have a causal relationship between them
B) both are correct, but there is no cause-and-effect relationship between them
C) the first is correct in itself, the second is wrong
D) The first is wrong, the second is correct in itself
E) both are wrong

A

ANSWER
D) The first is wrong, the second is correct in itself

EXPLANATION
The worsening of social status (position) is a very serious chronic stress condition leading to severe depression, impaired health status and an increased risk of early death in men according to the results of the „Hungarostudy” follow-up study.

263
Q

PBH - 385
Caries prevention by fluoridation of drinking water requires a political decision, because the fluoridation of drinking water can not be done at home, but only by the contribution of the water providers.
A) both are right, they have a causal relationship between them
B) both are correct, but there is no cause-and-effect relationship between them
C) the first is correct in itself, the second is wrong
D) The first is wrong, the second is correct in itself
E) both are wrong

A

ANSWER
A) both are right, they have a causal relationship between them

EXPLANATION
The fluoridation of drinking water in certain countries is based on the consideration that sufficient fluoride intake prevents tooth decay. The fluoridation of drinking water requires a political decision that can be made at local or national level, as technical changes are required by the water supplier. The fluoride content of the drinking water should be monitored continously to ensure that the fluoride content reaches the optimal level of 0.7 mg / L, but does not reach the maximum acceptable level of 4 mg / L, because that leads to an increased frequency of fluorosis.

264
Q

PBH - 386
There is no disease due to iodine deficiency in Hungary, because the consumption of iodised salt can prevent the formation of such diseases.
A) both are right, they have a causal relationship between them
B) both are correct, but there is no cause-and-effect relationship between them
C) the first is correct in itself, the second is wrong
D) The first is wrong, the second is correct in itself
E) both are wrong

A

ANSWER
D) The first is wrong, the second is correct in itself

EXPLANATION
The first statement is not true, the second is correct in itself. Low iodine content in drinking water could be seen in Hungary and surveys conducted in Hungary found that among school aged children the prevalence of goitre is 11% (4-23%). Based on urine iodine levels Hungary is considered as a country with a mild iodine deficiency. Iodised salt can be successfully used to prevent iodine deficiency. This programme was introduced in Europe in the 1920s, in developing countries in the 1990s. The aim is that at least 90% of households use iodised salt.

265
Q

PBH - 387
Vitamin D is considered as an essential nutrient because its chronic deficiency in children can lead to rachitis.
A) both are right, they have a causal relationship between them
B) both are correct, but there is no cause-and-effect relationship between them
C) the first is correct in itself, the second is wrong
D) The first is wrong, the second is correct in itself
E) both are wrong

A

ANSWER
D) The first is wrong, the second is correct in itself

EXPLANATION
The first statement is not true, the second is correct in itself. Vitamin D is considered as an important nutrient only when the ultraviolet sun rays entering the skin do not cause 7-dehydro cholesterol to produce enough vitamin D3. In urbanized environment of economically developed countries, infants, very young children and the elderly with some sort if impair in movement are at risk of lacking of vitamin D. In normal clothing, 15 to 30 minutes a day is usually enough to produce enough vitamin D in the skin due to UV irradiation. Professionals suggest vitamin D supplementation for infants and toddlers under age 3, since in their case the development and bone growth period is very intense and breast milk does not contain the adequate amount of vitamin D needed.

266
Q

PBH - 388
Vegetarian diet is usually not considered a healthy diet, because in the absence of animal proteins the essential amino acid requirements of the body can not be guaranteed.
A) both are right, they have a causal relationship between them
B) both are correct, but there is no cause-and-effect relationship between them
C) the first is correct in itself, the second is wrong
D) The first is wrong, the second is correct in itself
E) both are wrong

A

ANSWER
E) both are wrong

EXPLANATION
Vegetarianism is generally considered as a healthy diet - except for its extreme forms. Fruits, vegetables, legumes, whole grains, nuts and vegetable oils have a protective effect against cardiovascular and cancer diseases. Although most plant proteins are not complete, i.e. their amino acid composition does not meet the essential amino acid needs of humans, by a suitable combination of foods that the various incomplete protein sources complete each other (eg, combination of corn and beans can ensure an acceptable amino acid supply).

267
Q

PBH - 389
Although in the developed countries a much higher calcium intake could be seen, the number of hip fractures is higher, because high protein consumption increases renal calcium loss.
A) both are right, they have a causal relationship between them
B) both are correct, but there is no cause-and-effect relationship between them
C) the first is correct in itself, the second is wrong
D) The first is wrong, the second is correct in itself
E) both are wrong

A

ANSWER
A) both are right, they have a causal relationship between them

EXPLANATION
Among the nutritional risk factors of osteoporosis, inadequate calcium intake is one of the most important influencing factors. It is a fact that the incidence of hip fracture is significantly higher in developed than in developing countries, although in developed countries the calcium consumption is much higher. This is the so-called “Calcium paradox” caused by the fact that high protein consumption increases renal calcium loss. There is also a relationship between sodium and calcium that is excreted in the urine: if the diet contains less sodium, then less calcium is eliminated from the body.

268
Q

PBH - 391
In addition to limiting the intake of saturated fatty acids it is important in the prevention of cardiovascular diseases to consume whole-grain, vegetables, fruits and legumes, because a high-fiber diet has been proven to reduce the risk of ischaemic heart disease.
A) both are right, they have a causal relationship between them
B) both are correct, but there is no cause-and-effect relationship between them
C) the first is correct in itself, the second is wrong
D) The first is wrong, the second is correct in itself
E) both are wrong

A

ANSWER
A) both are right, they have a causal relationship between them

EXPLANATION
In addition to eating saturated fat, other dietary and lifestyle factors also have a significant impact on cardiovascular risk. Substitution of saturated fats in foods with carbohydrates, especially with refined cereal foods (foods with a high glycemic index), has been found to increase risk. However, replacing the saturated fats with unsaturated fats reduced the risk, this effect was probably supported by the higher consumption of plant foods ensuring higher intake of dietary fibers and micronutrients. In physically inactive and overweight populations, refined carbohydrates are likely to cause greater metabolic harm than saturated fatty acids. Therefore, dietary recommendations for the prevention of cardiovascular disease should be more focused on the consumption of whole grains, vegetables, fruits, legumes and on decreasing salt intake. If intake of saturated fats is reduced, it should be emphasized that the same (repleaced) percentage of energy intake has to be covered with unsaturated fatty acids and carbohydrates having low glycemic index.

269
Q

PBH - 392
Food safety is the responsibility of the workers of the food industry because it inot the responsibility of consumers to ensure food safety.
A) both are right, they have a causal relationship between them
B) both are correct, but there is no cause-and-effect relationship between them
C) the first is correct in itself, the second is wrong
D) The first is wrong, the second is correct in itself
E) both are wrong

A

ANSWER
E) both are wrong

EXPLANATION
Food safety is not only the responsibility of the food industry participants. The quality and safety of food is influenced by every single participant in the food chain (manufacturers/producers, processors, suppliers, food manufacturers, consumers). Consumers are also responsible for ensuring food safety at home.

270
Q

PBH - 393
The number of dental treatments for adults in Hungary has temporarily declined since 1995, because in 1995 a large number of dental treatments for adults were discontinued to be financed by state health insurance.
A) both are right, they have a causal relationship between them
B) both are correct, but there is no cause-and-effect relationship between them
C) the first is correct in itself, the second is wrong
D) The first is wrong, the second is correct in itself
E) both are wrong

A

ANSWER
A) both are right, they have a causal relationship between them

EXPLANATION
In 1995, the dental care support system was redesigned and since this time a variety of treatments for adult dental care, including dental preservation treatments, are offered in return for a fee. Since then, the number of people subjected to dental treatment has declined sharply, and many patients have had their teeth pulled out rather than opting for a chargeable conservation treatment. Recently, since 2000, he tooth conserving treatments were offered as charge-free treatments again.

271
Q

PBH - 394
The most important element of the reconstruction of the health-care system in the last years (and currently) has been the significant reduction in outpatient care, because outpatient care is the most expensive service in the healthcare sector.
A) both are right, they have a causal relationship between them
B) both are correct, but there is no cause-and-effect relationship between them
C) the first is correct in itself, the second is wrong
D) The first is wrong, the second is correct in itself
E) both are wrong

A

ANSWER
E) both are wrong

EXPLANATION
One of the main reasons of the reconstruction of healthcare systems is the considerable increase in health expenditures, which could not be covered by the amount of GDP available for the health care. Structural changes obviously affect primarily the most expensive services of the healthcare system. Accordingly, a significant reduction in inpatient care has happened in Hungary, as this is the most expensive element of the healthcare system. This process was necessarily linked to a strengthening of primary and outpatient care and the widest possible use of treatments that shortened the duration of hospital treatment (eg one-day surgery).

272
Q

PBH - 398
Equal opportunity is fundamental element of health care, because equal opportunities are not just related to health care, they also require multisectoralism.
A) both are right, they have a causal relationship between them
B) both are correct, but there is no cause-and-effect relationship between them
C) the first is correct in itself, the second is wrong
D) The first is wrong, the second is correct in itself
E) both are wrong

A

ANSWER
D) The first is wrong, the second is correct in itself

EXPLANATION
One of the conditions for equal opportunities is accessibility. In simplified manner accessibility means that a healthcare facility, hospital or emergency center is located nearby (within a certain distance or within a certain accessibility time), where the necessary care is available and can be used within a reasonable time. Equal opportunities mean more, it involves many factors that together ensure that people with the same health status actually have the same access to the necessary care. For example, the poor should not be limited by the fee of treatment to be paid in order to receive a proper treatment, or the family doctor should have an equal chance of sending people with the same illness to inpatient care. Equal opportunities also include patient’s information, because equal opportunities can only be achieved if patients have the same opportunity to recognize the need for treatment and to consult a doctor. Equal opportunities are therefore an extremely broad, cross-sectoral category defined by political, economic and other factors.

273
Q

PBH - 399
High-quality health services and equal opportunities even in developed countries can not be guaranteed for the whole population because healthcare providers never have a financial interest in reaching this situation.
A) both are right, they have a causal relationship between them
B) both are correct, but there is no cause-and-effect relationship between them
C) the first is correct in itself, the second is wrong
D) The first is wrong, the second is correct in itself
E) both are wrong

A

ANSWER
C) the first is correct in itself, the second is wrong

EXPLANATION
Equal opportunities in practice can not be guaranteed even in the richest countries, as there are always treatments (eg high technology, extremely high quality) that are expensive and can not be funded for the whole population. These costs may be is covered by supplementary insurance, while the richest part of the society receives the desired treatments under private care. Healthcare providers are keen to provide high-quality services, as this generate higher profits for them. Therefore, they are also interested in providing these services to as many people as possible.